Загадки для взрослых с: Загадки на логику для взрослых — сложные логические загадки с ответами

Загадки на логику для взрослых — сложные логические загадки с ответами

В мифах Древней Греции есть история о храбром и мудром Эдипе, который правильно ответил на загадку страшного и непобедимого Сфинкса, тем самым сохранив свою жизнь. Острый ум и смекалистость всегда ценились в людях, вне зависимости от эпохи, в которой они родились и жили. Предлагаем раскрыть свой потенциал с помощью подборки интересных загадок, среди которых есть как смешные, так и сложные или с подвохом, поэтому будьте внимательны и не спешите сразу давать ответ.

Сложные логические загадки для взрослых

1. «Необычные цены»

Мужчина зашел в хозяйственный магазин и спросил цену на один товар. Продавец ему ответил:

  • один стоит 1 $;
  • восемь — 1 $;
  • тринадцать — 2 $;
  • сто шестьдесят восемь — 3 $;
  • четыре тысячи девятьсот восемьдесят шесть — 4 $.

Что покупал мужчина?

Показать ответ

Номерки на двери.

2.

«Свет»

Вы стоите перед закрытой комнатой. Внутри помещения есть 1 лампочка накаливания. Окна и щели, чтобы разглядеть свет внутри комнаты, отсутствуют.

Справа от двери ведущей в комнату установлено 3 переключателя, которые находятся в положении «выкл», но только один из них подключен к лампе. Нужно узнать, какой переключатель включает лампочку, но при этом зайти в комнату можно только 1 раз.

Примечание: менять положение переключателей разрешено неограниченное количество раз.

Показать ответ

Включите сразу два любых переключателя, а один оставьте в положении «выкл». Подождите 2 минуты, выключите один из включенных переключателей и сразу войдите в комнату:

  • если лампа горит, значит, это тот переключатель, который остался включенным;
  • если света в комнате нет, потрогайте лампочку: горячая — рабочий переключатель, который был выключен перед входом в помещение, холодная — подключен переключатель, положение которого ни разу не менялось.

3. «Строитель и кирпичи»

У строителя есть 8 кирпичей, 7 из которых имеют одинаковый вес, а один — тяжелее. Как строителю выявить бракованный кирпич с помощью весов всего за два взвешивания?

Показать ответ

Все нужно делать поэтапно:

  1. Разделить 8 кирпичей на три стопки по 3, 3 и 2 штуки.
  2. Взвесить 2 стопки, в которых по 3 кирпича.
  3. Если они уравновесились, значит взвесить стопку из двух кирпичей и найти бракованный.
  4. Если не уравновесились, то взять 3 кирпича из стопки, которая перевесила и разделить на 3 штуки.
    Положить по одному на весы. Если один перевесил — он бракованный, если уравновесились, значит, тяжелее всех тот, что остался в стороне.

4. «Блюдо для Фараона»

Повару древнего Египта необходимо варить яйцо в кипящей воде строго две минуты, но у него в наличии песочные часы только на три, четыре и пять минут. Как ему приготовить блюдо Фараону, не отклоняясь от времени, которое указано в рецепте?

Примечание: использовать часы можно на свое усмотрение: сразу все, только одни, по очереди или по несколько раз.

Показать ответ

Когда вода закипит, повару нужно поставить двое часов: на три минуты и на пять. Когда песок в трехминутных часах закончится, сразу опускать яйцо, таким образом, оно будет вариться требуемое время.

5. «Женский возраст — это тайна»

Парень спросил у девушки: «Сколько тебе лет?», на что она загадочно улыбнулась и ответила: «Позавчера мне было 22, а в следующем году будет 25».

Попробуйте догадаться, когда у девушки День рождения и какого числа происходил разговор между парой?

Показать ответ

Парень и девушка разговаривали 1 января, а День рождения был 31 декабря. Получается:

  • «позавчера» было 30 декабря и девушке было 22 года;
  • 31-го числа ей исполнилось 23;
  • в День рождения, который будет в тот год, когда велся разговор, исполнится 24;
  • а на следующий год девушка отметит 25-летие.

6. «Загадочный треугольник»

Каким образом нарисовать треугольник, чтобы все его углы были 90 градусов?

Показать ответ

На мяче или любом другом сферическом предмете.

7. «Мудрый отец»

Сын попросил у отца 10 000 долларов на обучение, на что тот ответил, что не может ему дать их просто так, поскольку не уверен в его способностях и в том, что он сможет закончить экономический факультет. Но предложил пари.

Если сын разложит 1000 долларов по 10 конвертам таким образом, чтобы какую бы сумму отец не назвал, парень мог дать ему это количество денег одним или несколькими конвертами, но при этом не доставая и не докладывая деньги внутрь, то он получит 10 000 долларов.

Сын пришел через 30 минут с 10 запечатанными конвертами и отец назвал сумму 555 долларов, на что парень ему выдал три конверта. Открыв, отец пересчитал все деньги и в сумме оказалось ровно 555 долларов, после чего выписал чек на 10 000 долларов ему на обучение. Каким образом сын разложил деньги по 10 конвертам, чтобы можно было получить любую сумму до 1000 долларов включительно?

Показать ответ

Парень разложил в конверты такие суммы в долларах: 1, 2, 4, 8, 16, 32, 64, 128, 256, 489.

8. «Простое добавление»

Если одиннадцать плюс два равняются одному, чему равны девять плюс пять?

Показать ответ

Речь идет о времени: 11 часов плюс 2 часа = 1 час, а 9 часов плюс 5 часов = 2 часа.

9. «Восемь восьмерок»

Запишите восемь восьмерок таким образом, чтобы в сумме получилась тысяча.

Показать ответ

888 + 88 + 8 + 8 + 8 = 1000.

10. «Две нити»

У вас есть две одинаковые по длине нити, и известно, что при зажигании одного конца любой из них горение будет длиться ровно один час. Как с помощью всего двух таких ниток и спичек отмерить 45 минут, не прикасаясь при этом руками к ниткам?

Показать ответ

Зажгите оба конца первой нити и один конец второй. Когда полностью сгорит первая нить, это будет сигнализировать о прошествии 30 минут. Сразу же подпалите второй конец горящей нитки таким образом, чтобы огонь шел навстречу друг другу. В результате, когда она полностью сгорит, пройдет 45 минут.

Смешные загадки на логику

Как проходит разминка? Все получилось разгадать? Уверены, что да! А теперь попробуйте потренироваться на более веселых и смешных загадках, которые также могут стать отличным развлечением среди друзей большой компании.

1. «Несмелый ковбой»

Ковбой вбегает в бар и кричит: “Срочно мне налейте стакан воды!” Вместо этого бармен выхватывает пистолет и стреляет в воздух. Ковбой испугался, поблагодарил бармена и вышел. За что ковбой остался благодарен бармену?

Показать ответ

У ковбоя была икота, и когда бармен выстрелил, от испуга она прошла.

2. «Злостный нарушитель»

У женщины не было при себе водительского удостоверения. Она проехала без остановки на железнодорожном переезде при опущенном шлагбауме. Потом, проигнорировав «кирпич» двинулась во встречном направлении по улице с односторонним движением и остановилась только через три перекрестка. Сотрудники ГИБДД, видя все это, решили не вмешиваться. Почему?

Показать ответ

Женщина ехала на электричке, а потом шла пешком.

3. «Яркая профессия»

Олег зашел в магазин купить колбасы. Когда очередь дошла до него, он попросил порционно нарезать палку, при этом не поперек, а исключительно вдоль. Продавец поинтересовалась, не пожарником ли он случайно работает, на что Олег ответил: «Да». Как женщина догадалась о его профессии?

Показать ответ

Правильный ответ. Олег был в форме и вышел в обеденный перерыв.

4. «Практичные приметы»

Кому и при каких условиях не повезет встретить черную кошку «лицом к лицу»?

Показать ответ

Мыши. При любых условиях.

5. «Спортивный Санта»

Почему Санта Клаус так хорош в карате?

Показать ответ

Потому что у него черный пояс.

6. «Милые мишки»

Как называется медведь, у которого нет зубов?

Показать ответ

Мармеладный мишка.

7. «Слезы змеи»

Как заставить расплакаться гремучую змею?

Показать ответ

Забрать у нее погремушку.

8. «Конфуз в блошином цирке»

Почему собаку выгнали из блошиного цирка?

Показать ответ

Пес украл всех артистов шоу!

9. «Неожиданная поза»

Что происходит с курицей, когда она лежит вверх ногами?

Показать ответ

Ее приготовили и едят.

10. «Странная схожесть»

Что общего между ежиком и молоком?

Показать ответ

Они оба могут сворачиваться.

Логические загадки для взрослых с подвохом

А теперь попробуйте мыслить нестандартно и креативно, стараясь заметить мелочи и не повестись на уловки, ведь далее представлена подборка загадок с подвохом.

1. «Непонятный возраст»

Одной женщине в 1850 году исполнилось 40 лет, но в 1860 году ей было 30 лет. Как такое могло получиться?

Показать ответ

В 1850 году женщине только исполнилось 40 лет, в то же время в 1860 году ей уже было 30, получается 1860 год был раньше чем 1850, а такое возможно только в том случае, если летоисчисление ведется до рождества Христова, то есть 1850 год до н. э. и 1860 год до н. э.

2. «Игра до рассвета»

Четверо мужчин сели играть. Игра длилась всю ночь до самого рассвета. Причем все они играли за деньги, а не ради удовольствия. Когда пришло время сводить счета, мужчины заработали приличные суммы и ни один из них не проиграл.

Как такое могло выйти?

Показать ответ

Эти мужчины — члены оркестра, нанятого для частной вечеринки.

3. «Правила дорожного движения»

Перед перекрестком на светофоре стоят и ждут зеленого света: фура, телега с лошадью и мотоциклист. Загорелся желтый, водитель фуры нажал на газ, отчего лошадь испугалась и укусила за ухо мотоциклиста. Кто из троих нарушил ПДД?

Показать ответ

Мотоциклист, так как был без шлема.

4. «Человек дождя»

Художник гулял по парку, когда на улице начался дождь. С собой у мужчины не оказалось шляпы и зонтика, а в кронах деревьев из-за сильного ливня укрыться не вышло. В результате вся одежда оказалась влажной, но ни один волос на голове художника не промок.

При каких обстоятельствах такое могло произойти?

Показать ответ

Это был лысый художник.

5. «Провинившийся провидец»

Находясь в лагере, развернутом на поле во время военных действий, солдат был в ночном карауле. Около 4-х часов утра он разбудил командующего и сообщил, что этой ночью увидел во сне, как противники атакуют их лагерь с юга. И хотя командующий не был суеверным человеком, все же выставил наблюдателей за южной частью равнины.

Спустя некоторое время, противник напал именно с указанной солдатом стороны, но так как меры были предприняты заранее, войско смогло с легкостью отбить атаку и напасть в ответ на противника, победив его. После изнурительного боя и одержанной победы командующий поблагодарил солдата и посадил его в тюрьму. Почему?

Показать ответ

Сны человеку снятся, только когда он спит, а караульному это запрещено.

6. «Смекалистый отличник»

Перед началом экзамена профессор объявил студентам: «На этом письменном экзамене есть три вопроса. У вас будет всего один час, чтобы ответить на все, и не более. Любой, кто будет писать дольше, получит двойку без права на пересдачу!».

Андрей, получив свой билет, принялся отвечать. После того как пропищал таймер, осталось всего 5 минут, студент понял, что ответил только на два вопроса и часть третьего. Осмотревшись в аудитории, парень заметил, что с их группой сдает другой экзамен старший курс, а значит, ему может сойти с рук несколько лишних минут, пока другие будут сдавать работы.

Закончив отвечать на вопрос, Андрей пошел сдавать работу, но по дороге его перехватил профессор и сказал: «Я видел, как после завершения одного часа, ты продолжал нагло писать, а потому заслуживаешь только двойку!»

Парень подумал и спросил: «Вы знаете, кто я?» На что профессор покачал головой, подтвердив, что даже не имеет представления, кто он такой, и ответил: «Меня не волнует, кто ты такой. А вот тебе нужно уважать меня и соблюдать дисциплину!»

– Хорошо! — ответил Андрей, положил работу и ушел. В день объявления результатов он получил оценку «отлично». Почему?

Показать ответ

Когда профессор подтвердил, что не знает, кто такой Андрей, тот быстро сунул экзаменационную работу в общую стопку на столе и ушел, чтобы профессор не знал, какая работа именно его и заслуживает неудовлетворительной оценки за несвоевременную сдачу. Поэтому во время проверки это требование профессор просто не учел и оценивал лишь знания студента, которые показали отличный результат!

7. «Знаменитости»

Наполеон Бонапарт, Петр Первый, Юрий Гагарин, Шерлок Холмс, Вильям Шекспир, Нострадамус, Пифагор. Кто лишний в этом списке?

Показать ответ

Шерлок Холмс, поскольку это вымышленный персонаж, а остальные — реальные люди.

8. «Восемь подружек»

Восемь подружек находятся в доме и каждая из них занята каким-то делом:

  • первая — готовит ужин;
  • вторая — разжигает камин;
  • третья — читает книгу;
  • четвертая — играет в шахматы;
  • пятая — рисует картину;
  • шестая — убирается в доме;
  • седьмая — вешает шторы.

Чем занята восьмая девочка?

Показать ответ

Играет в шахматы вместе с четвертой подружкой.

9. «Прогноз погоды»

Если в 12 часов ночи пошел снег, можно ли предположить, что через 96 часов на улице будет солнце?

Показать ответ

Такого быть не может так как 96 часов — это ровно четверо суток, а значит, через указанный промежуток времени снова будет 12 часов ночи.

10. «Четыре сестры»

У мамы Мэри четыре дочери. Имя первой — Априна, второй — Мая, третьей — Июния.

Как зовут четвертую девочку?

Показать ответ

Ее зовут Мэри.

Математика и логика для детей 7-13 лет

Развиваем логическое мышление через решение сюжетных математических задач в интерактивном игровом формате

узнать подробнее

Загадки для взрослых

Взрослые тоже любят отгадывать загадки.
На этой странице вас ждет подборка загадок повышенной сложности.

Что за кушанье, что всякий им питался, Но на стол оно не подается?

грудное молоко

Два соболя Лежат хвостами Друг к другу.

брови

Узловат Кузьма! Завязал узла — Развязать нельзя.

пупок

Мужик наземь кидает, Барин в карман собирает.

сопли

Лежит колода Поперек дорога; В колоде двенадцать гнезд, В гнезде по четыре яичка, В яичке по семи зародышков. Что выйдет?

год

Выше лошади, ниже собаки.

седло

Висит, болтается, Все за него хватаются.

полотенце

Я не сам по себе, А сильнее всего И страшнее всего, И все любят меня, И все губят меня.

огонь

В темном бору Сидит птица; Всяк ее боится, Никто от нее не уйдет: Ни царь, ни царица, Ни красная девица, Ни рыба в море, Ни заяц в норе.

смерть

Курят дяди, курят тети, Курят дома, на работе, А вокруг все говорят, Что табак всем людям -…

яд

В лес дорога. На пупке тревога. Внутри ярмарка.

улей

Одного не знаю, Другого не вижу, Третьего не помню.

смерть, возраст, рождение

Заря-заряница, красная девица, По лесу ходила, ключи обронила. Месяц видел — не сказал, Солнце увидало — подняло.

роса

Живёт без тела, Говорит без языка; Никто его не видит, А всякий слышит.

эхо

Толстый тонкого побьет. Тонкий что-нибудь прибьет.

молоток и гвоздь

Бывает ночная, Бывает гавайская, Мужская и женская, Даже китайская. Родиться в ней можно, А также жениться, На службу сходить И с друзьями напиться.

рубашка

Если ты хочешь, Чтоб праздник был ярок, Нужно купить для любимой. ..

подарок

Если сахарной станет она, То на празднике детям нужна. Медицинской она назовётся, Без неё доктор не обойдётся. А строители, кроя всех матом, Минеральную выберут…

вату

Любой здесь мужик отдыхает душою. Хоть помещение и не большое, Здесь пиво, друзья, рядом шиномонтаж Отдушина эта зовётся…

гараж

Чтобы в углу они не стояли, И запахом всех наповал не сражали, Нужно их вечером в стирку кидать, А утром другие себе надевать.

носки


🔎 Загадки для другого возраста

Смешные загадки с подвохом для детей и взрослых (с ответами)

Смешные загадки с подвохом, разгадывание кроссвордов, шарад является любимейшим времяпровождением миллионов людей по всей России. Это очень хорошая тренировка для ума. Популярным это занятие стало ещё в годы жизни наших дедов. Появление Интернета увеличило интерес к ним, потому что найти интересную загадку стало проще.

Очень интересны шарады для детей. Развивать ум ребёнка, его находчивость важнее, чем взрослому человеку. До 7 лет это занятие является обязательным элементом воспитания ребёнка. В раннем школьном возрасте главным становится выполнение уроков, поэтому школьников важно не перегружать. Взрослому человеку это занятие тоже необходимо, чтобы не терять находчивость и уметь принимать быстрые правильные решения в сложных ситуациях.

Детские смешные загадки с подвохом даются не всем взрослым (с ответами)

Чтобы вызвать у детей интерес, им лучше задавать смешные загадки. Это позволит им не только напрячь ум, но и разлечься. Для дошкольника это обязательный элемент развития. Психологически ему трудно настроиться на что-то серьёзное.

Смешные загадки с подвохом это необычные загадки. Они, представляют из себя вопрос, на который следует быстро ответить в рифму. Он всегда неправильный. Это позволяет развить у детей чувство юмора, приучить, не воспринимать на веру всё, что говорят взрослые. Это очень сильно развивает мышление ребёнка. Ведь, если ответ не правильный, до правильного нужно догадаться самому, да ещё и распознать сам факт ошибки. Это очень помогает в жизни.

Простые примеры детских загадок с подвохом

? По лесу бежит 50 волков. Сколько у животных хвостов на шее? Ответ 0, хотя логика требует ответа 50. Причина в том, что хвост на шее не растет. Не каждый взрослый разгадает сразу эту загадку.

Очень важно попросить ребёнка представить стаю животных с хвостами на шее. Насколько это вообще можно сделать. Это не фильм ужасов. Так ребёнок догадается о правильном ответе и увидит, что первоначальные чувства его обманули.

? — Магазин охраняет сторож. На его голове сидит воробей. Чем занимается сторож? Ответ: спит.

? — Можно или нет урезать слово «мышеловка» так, чтобы оно стало состоять всего из 5 букв. Ответ: Да, можно, и это будет кошка.

? — Чем больше ковыряешь, тем больше становится? Это дырки.

? — Какие часы показывают верные цифры только 2 раза в день? Стоячие.

? — Существует ли в мире, конь, который не питается овсом? Да, шахматная фигура.

? — Есть ли в машине колесо, которое не участвует в движении? Да, запаска.

? — Существует ли узлы, которые нельзя развязать? Да, железнодорожные. 

Детям очень понравятся такие загадки. Они привлекут их возможностью развлечься, не задумываться над ответом. Ведь, он не лежит на поверхности. Довести до психики всю сложность и важность этих упражнений задача воспитателя. Это не очень сложно сделать. Достаточно поиграть с ребёнком в игру с использованием нескольких загадок.

Смешные загадки с подвохом для взрослых 

Смешные загадки с подвохом популярны не только у детей, но и у взрослых. Их необходимо разгадывать в качестве зарядки для ума. Их ценность ниже, чем у детей, но суть остаётся прежней. Это позволяет человеку расслабиться, узнать что-то новое, смотреть на жизнь с юмором.

Загадки это развлечение, которое позволяет погрузиться в другой мир, это извлечение человека из мира реального в мир комедии и сатиры.

Задайте эти загадки детям, может быть получите правильный ответ быстрее остальных.

Примеры загадок:

? — Висит груша, но её нельзя сорвать или съесть. Такое вообще возможно и если возможно то где? Да, в тренажёрном зале. Там висит боксёрская груша, ее съесть нельзя.

? — Существует ли что-то съестное, что нельзя съесть на завтрак? Да, то, что ты приготовил на обед, ужин или полдник.

? — Ни рук, ни ног, но на женщину залезает. Коромысло.

? — Существует ли человек, который не намочит волосы во время ливня? Да, лысый.

? — Есть ли нечто, что легко поднять, но трудно метнуть? Пух.

? — Предположим, что есть пять берёз. На них растут длинные ветки, на тех ещё маленькие, а на маленьких яблоки. Сколько растёт яблок? 0, потому что на берёзах яблоки не растут.

Загадки для взрослых сложнее, чем для детей, они более научные, информативные и сложные. Подвох, однако, есть во всех.

Интересный пример:

? — Кто если войдёт в Кремль, уже никогда оттуда не выйдет? Жириновский.

? — Кто пугает людей кровью и броневичком? Зюганов.

Такие загадки уже скорее политические, но юмора и подвоха в них, даже больше, чем надо. Много хороших загадок обыгрывают вечные темы. Это может быть спор зятя с тёщей или снохи со свекровью. Любопытные загадки на тему мужчины-женщины, или муж-любовник, жена-любовница. Юмора там может быть очень много. Речь идёт о нормальных загадках, лишённых злобной ненависти и грубости.

? — Одеждой не является, но всё время обитает в дамском шкафу? Любовник.

Сложные загадки для взрослых на логику и сообразительность

Загадки могут быть более сложными, чтобы их разгадать придётся включить логику:

? — Господа, назовите домашнее животное, которое начинается с буквы «Т»? Таракан.

? — Есть три тракториста. У них есть брат Пётр. У Петра нет братьев трактористов? Возможно ли такое? Да, если трактористы женщины!

? — В комнате есть 50 свечей. 20 из них задули. Сколько свечей останется в комнате через 5 часов. 20, т.е. те, что задули, потому что остальные сгорят.

? — Сколько перьев заходит в ворота? Нисколько. Потому что перья не ходят.

? — Допустим, что Вам подарили на день рождения линейку, карандаш, ластик и циркуль. Перед Вами стоит задача нарисовать окружность. С чего вы начнёте? Придётся начать с того, что где-то найти бумагу, потому что без неё этот подарок будет бесполезным.

? — Идут по саду два отца и сына. Видят, на дереве растут три апельсина. Сорвали и решили поделить. Получилось, что всем досталось по апельсину. Возможно ли такое? Да, если по саду шли дед, отец и сын.

? — До работы лежит, во время работы стоит, а после неё влажный? Зонт.

? — В воду упали два гвоздя. А, дальше с ними случилось нечто… то, что является грузинской фамилией. Что это? Заржавели.

? — Существует ли посуда, откуда невозможно ничего съесть? Да. Пустая.

? Чаепитие

  1. Три чашки и десять кусочков сахара. Разложите кусочки в чашки таким образом, чтобы в каждой было нечетное число.
  2. Как разложить в три чашки десять кусочков сахара поровну?

Загадки тренируют ум, помогают развить полученные в детстве навыки, являются лекарством от застоя в уме. В тоже время они спасают людней от депрессии, и помогают держать в тонусе психику. Их разгадка это очень полезное занятие, которое тренирует мозг.

Желаю Вам приятных отгадок интересных загадок. 

 

интересные статьи

Забавные загадки для застолья с ответами.

Советы по организации праздника

Веселые праздники любят взрослые и дети. Но чтобы вечеринка не превратилась в скучное поедание вкусностей, нужно составить развлекательную программу. В перерывах между приемами пищи гости смогут поучаствовать в конкурсах, проявить себя в забавных играх. Но главное – это прикольные загадки для застолья! После подвижных игр приглашенные друзья и близкие с удовольствием поработают мозгами. Ведь загадки – это лучшая зарядка для ума.

Пир на весь мир

Семейное торжество – это грандиозное событие. Подготовиться нужно тщательно: продумать декор помещения, меню и программу развлечений. Прикольные загадки для застолья с ответами запишите на листе бумаги. Приготовьте побольше каверзных вопросов на случай, если гости не захотят принимать участие в конкурсах. А вот пошевелить мозгами никто не откажется! Можно объединить загадки по темам: «животные», «еда», «имена», «предметы». Или, наоборот, задавать их вразброс, чтобы запутать публику.

Взрослая компания

Если за столом присутствуют взрослые и дети, нужно подготовить задания для обеих групп. Малыши тоже с удовольствием подумают над ответами. Начните с легких вопросов, постепенно их усложняя. Вот некоторые прикольные загадки для взрослых.

Зерно, которое прошло огонь, медные трубы и воду (самогон).

Почему у Санта-Клауса нет Снегурочки? (он доберется до дома самостоятельно, а вот Деда Мороза должен кто-то дотащить).

Как назвать тень от листьев хрена? (хренотень).

Почему у козы всегда печальные глаза? (потому что муж – козел).

Как поймать тигра в клетку? (тигр в полоску).

В женской сумочке есть все, кроме… (порядка).

Какое самое длинное слово в русском языке? («Слово о полку Игореве»).

Такие прикольные загадки для застолья оценит любая компания!

Талант

Устройте гостям настоящее шоу! Если вы обладаете прекрасным слухом и голосом, можно пропеть загадки в ритме частушек. Подготовьте русский народный костюм: сарафан и кокошник или повяжите платок на голову. Появитесь в комнате эффектно, под музыку, с улюлюканьем и криками. Приглашенные придут в полный восторг от такого зрелища! Прикольные загадки для взрослых, пропетые вами, будут немедленно разгаданы.

Одно яйцо сварится за три минуты. Сколько понадобится времени, что сварить три яйца? (3 минуты).

От большого квадратного стола отпилили один угол. Сколько теперь углов осталось у стола? (пять).

Почему у гориллы такие огромные ноздрищи? (потому что пальцы у нее такие же).

В общественном месте эта женщина сначала трется вокруг вас, а затем требует деньги! Кто она? (кондуктор).

Под проливным дождем у него не мокнут волосы! Кто это? (лысый человек).

Такие прикольные загадки для застолья будут уместны на любом мероприятии.

Рифма

Дети тоже не прочь подумать над сложными вопросами. Подготовьте развлечение и для них. Обязательно приобретите небольшие призы, ведь малыши будут ждать поощрения. Хорошо подойдут канцелярские товары, конфеты, небольшие игрушки и сувениры. Можно устроить состязание между двумя командами. Пусть ребята посоревнуются в смекалке и сообразительности. Хорошо впишутся в программу прикольные загадки для застолья в стихах. Читать их надо медленно и четко, чтобы дети хорошо расслышали.

Чтоб зимой не мерзли ноги, чтобы бегать по дороге,

И взрослые люди, и дети маленькие одевают… (валенки).

А модной дочурке и супруге папа купит… (угги).

Почтальон несет к нам в дом, не книгу и не альбом,

Обо всем, что случилось где-то, расскажет нам… (газета).

Некрасивый я на вид, но совсем не ядовит,

На игольницу похож, я лесной и серый… (еж).

Моя подруга – это нить,

Мы можем все на свете сшить,

Я тонка и колка, зовут меня… (иголка).

Оно на вкус не как вода,

Бело как снег оно всегда,

Продают на рынке, в пакете и в бутылке! (молоко).

Во дворах и там и тут эти красавицы цветут,

Почти до самого мороза наш взгляд балует… (роза)!

Он пузатый, вкусный, гладкий,

И растет на каждой грядке,

Можно в салат его положить, в бочке можно засолить. .. (огурец).

Такие прикольные загадки для застолья в стихах смогут одолеть и малыши, и дети постарше. К ним обязательно подключатся и взрослые, ведь стоит только начать давать правильные ответы и удержаться от азарта и не выкрикнуть первым просто невозможно!

Развиваем интеллект

Загадки действуют благотворно на людей любого возраста. Если за весь вечер гость не смог дать ни одного правильного ответа, он обязательно задумается! Развивать интеллект не поздно никогда. Читайте больше книг, уделяйте время своим малышам! Пусть они сами придумывают каверзные вопросы и задают их вам. Можно ввести новую семейную традицию – час загадок после ужина. Это пойдет на пользу всем членам семьи. Записывайте прикольные загадки для застолья и радуйте ими своих близких и друзей. Так можно собрать «золотую» коллекцию, которая не будет иметь аналогов, а также заработать репутацию хорошего организатора.

Устраивайте яркие праздники и веселитесь от души!

Загадки для взрослых логика.

Взрослые загадки с подвохом. Загадка как способ развития ребенка

Большая подборка загадок с ответами для самых забавных и интеллектуальных развлечений с детьми. Погружаясь в «загадочный» мир, где пересекаются фантазии и реальность, взрослые забывают обо всем на свете и превращаются снова в детей, а дети учатся находить верные решения, скрытые в самых обычных словах. Не секрет что детские загадки помогают развитию вашего ребенка.

Загадки для детей бывают разными. Можно проводить домашние викторины по праздникам и выходным, в семейном кругу. В такие мероприятия можно включать загадки про еду, животных, птиц и насекомых, а также загадки-обманки.

Кто-то из знаменитых психологов нашей современности сказал, что загадки заставляют думать. Еще бы! Загадка, или небольшое веселое четверостишье, которое объясняет слово, влияет на формирование мировоззрения ребенка, развивает интеллект, логику, воображение и внимание, скорость реакции.

Как выбирать детские загадки?

Детские загадки должны быть увлекательными, смешными, немного сложными, чтобы ребенок не сразу догадался. Детям свойственно любопытство. Им понравятся загадки для детей на сообразительность. Не стоит до последнего принимать поражение ребенка, даже если он просит выдать ответ. Лучше предложите ему приз, если он разгадает загадку сам.

За гранью серьезности, среди зашифрованных ответов, тайн и секретов, начинается игра воображения, в процессе которой развивается логическое, ассоциативное и творческое мышление детей.

Конечно, родителям не всегда хочется утруждать драгоценное чадо и придумывать для него наисложнейшие загадки. Тем не менее такие вопросы, над ответом на которые нужно поразмыслить, полезны и нужны для детей и взрослых, независимо от возраста.

Зачем загадывать ребенку сложные загадки

Мамы и папы могут задаться вопросом о том, стоит ли вводить ребенка в заблуждение и включать в программу непростые задачки. Тем не менее, изучив информацию о том, насколько продуктивны наисложнейшие загадки для детей разных возрастов, родители тут же изменят свое былое мнение. Загадки на логику и с подвохом нужны по следующим причинам:

Это лишь некоторые факторы, которые указывают на то, что для детей обязательно нужны непростые вопросы, на которые надо найти ответы. Это поможет полноценно развиваться и быть грамотным.

Какими должны быть загадки

Понятное дело, что сложные загадки несколько отличаются от простых логических вопросов. Продумать программу развивающего занятия с такими заданиями следует предварительно, чтобы процесс прошел с легкостью и без заминок. Наисложнейшие загадки должны быть:

  • С подвохом.
  • Двусмысленны.
  • Такими, над ответом на которые стоит сильно задуматься.
  • Сложные загадки должны быть подобраны по возрасту ребенка. Это поможет мальчикам и девочкам найти ответы по уровню знаний. Отсюда следует, что малышам не стоит загадывать очень сложные загадки, для самых маленьких лучше подобрать вопросы с подвохом. Для деток же постарше можно выбирать такие вопросы, как для взрослых.

Стоит учитывать вышеуказанные факторы, подбирая логические вопросы для своего чада.

Загадки на логику для самых маленьких

Для деток дошкольного возраста можно взять во внимание следующие загадки:

На березе росло три яблока, а на тополе — пять груш, сколько всего фруктов на этих деревьях?

(Ни одного, на березе и тополе не растут фрукты)

Как можно найти черную кошку в темной комнате?

(Включить свет)

Каким станет красный с белой вышивкой платочек, если его опустить в Черное море?

Чего нельзя съесть на обед?

(Завтрак и ужин)

Что будет в следующем году с собакой, которой исполнилось пять лет?

(Ей будет шесть лет)

Чьи волосы не намокнут под проливным дождем?

(Лысого человека)

Как правильнее сказать: не видно белого желтка или не вижу белый желток?

(Никак, желток не бывает белым)

Утка, стоящая на одной лапе, весит три килограмма, сколько будет весить эта же утка, если она станет на две лапы.

(3 килограмма)

Два яйца варятся 4 минуты, сколько будут вариться десять яиц?

(4 минуты)

Возле лавочки отдыхает кошка. И хвост, и глаза, и усы — все, как у кошки, но это не кошка. Кто отдыхает возле лавочки?

Угадай, что пропадет, если съесть бублик?

Как можно зажечь спичку, когда находишься под водой?

(Можно, если находишься в подводной лодке)

В зале было зажжено 30 свечей. Зайдя в комнату человек, погасил 15 из них. Сколько свечей осталось в зале?

(Осталось 30 свечей, затушенные свечи по-прежнему в комнате)

У дома неровная крыша. Одна сторона опущена больше, другая меньше. Петух сел на вершину крыши и снес яйцо, в какую сторону оно покатится?

(Никуда не покатится, петух не несет яйца)

Под какое дерево прячется лисица во время дождя?

(Под мокрое)

На каких полях не растет ни одного растения?

(На полях шляпки)

Такие сложные загадки на логику для самых маленьких вызовут водоворот эмоций и интереса. Самое главное, давать ребенку намеки, благодаря которым он сможет найти правильный ответ.

Сложные загадки с подвохом для школьников

Детям школьного возраста можно подобрать вопросы еще труднее. Очень сложные могут быть следующими:

Ты на соревновании по бегу. Когда ты перегнал того, кто бежал последним, каким ты стал?

(Такого не может быть, ведь последнего бегуна нельзя перегнать, потому что он последний и сзади него не может быть еще кого-то)

У троих владельцев автомобилей был брат Алеша. Но у Алеши не было ни одного брата, как такое возможно?

(Возможно, если у Алеши были сестры)

Каким вы станете по счету, если перегнали второго по очереди бегуна?

(Многие ответят первым, но это неправильно, потому что перегнав второго бегуна человек станет вторым)

Такие сложные загадки с подвохом непременно понравятся школьникам. Поразмыслив над ответом, его будет несложно озвучить.

Взрослые загадки с подвохом

Порой взрослые люди, словно дети. Поэтому очень сложные загадки им тоже придутся по душе. Людям старше школьного возраста можно задать следующие логические вопросы:

Едет трамвай с пятью пассажирами. На первой остановке вышли два пассажира, зашли четыре. На следующей остановке никто не вышел, зашли десять пассажиров. Еще на одной станции вошли пятеро пассажиров, один вышел. На следующей — вышли семеро, зашли восемь человек. Когда была еще одна остановка, вышли пятеро и никто не заходил. Сколько всего остановок было у трамвая?

(Ответ в данной загадке не столь важен. Суть в том, что все участники, скорее всего, будут считать количество пассажиров и вряд ли кто-то примет решение подсчитывать остановки)

В дверь звонят. Ты знаешь, что за ней находятся твои родственники. В твоем холодильнике стоят шампанское, холодная вода и сок. Что ты откроешь в первую очередь?

(Дверь, потому что гостей сначала нужно впустить в квартиру)

Здорового человека, который не болеет, не имеет инвалидности и у которого все в порядке с ногами, выносят на руках из больницы. Кто это?

(Новорожденный ребенок)

Зашел ты в комнату. В ней находятся пять котов, четыре собаки, три попугая, две морские свинки и жираф. Какое количество ног стоит в комнате на полу?

(На полу две ноги. У животных лапы, ноги только у человека)

Трое заключенных, не зная запланировали сбежать из тюрьмы. Тюрьму окружала речка. Когда сбегал первый заключенный, на него напала акула и съела его. Так погиб первый из сбегающих. Когда второй заключенный осуществлял попытку бедствия, его заметили часовые и за волосы потащили на территорию тюрьмы, там его расстреляли. Третий заключенный убежал нормально, и больше его никто не видел. Что не так в этой истории?

(Акул в речке не бывает, заключенного не могли потащить за волосы, потому что их бреют наголо)

Такие загадки понравятся взрослым участникам мероприятия.

Как мотивировать ребенка к участию в развивающем занятии

Понятное дело, что детям обязательно нужна мотивация для того, чтобы участие в игре было азартным и желаемым. Достаточно просто пообещать ребеночку какой-то презент и, конечно же, вручить его в конце игры.

В этом разделе представлены самые интересные загадки для взрослых разной тематики – смешные, на смекалку, с подвохом, в стихах, пошлые, про секс. Загадки для взрослых хороши к любому случаю. Отлично подойдут для веселой компании, особенно когда гости уже устали поглощать салаты и танцевать, можно сидя за столом продолжить веселье. Загадайте веселые загадки с подвохом для своих гостей и продолжение банкета обеспечено! Школьники и студенты также смогут подобрать материал для своей вечеринки. Попробуйте загадать одну из загадок нашего сайта своему другу или подруге — хорошее настроение гарантировано и вам и вашим друзьям! Загадки для взрослых с подвохом помогут Вам не просто разыграть друзей, но и поржать от души над ответами. Загадки с ответами хорошо подходят для проведения конкурсов по разгадыванию загадок на праздничных мероприятиях или просто весело провести время с друзьями. Данный раздел Загадки для взрослых предназначен для того, чтобы улучшить настроение взрослых, дать им возможность немного поразмыслить, в целом весело провести время!

Мы ребята удалые, лезем в щели половые.
Тараканы

Я к жене пристать хотел, Только «друг мой» не хотел.
Думал, что инфекция, Оказалось, вот позор, Потерял…

эрекцию

Волосатая головка за щеку заходит ловко.
Зубная щетка

Выше колена, пониже пупка, дырка такая, что влезет рука.
Карман

В темной комнате, на белой простыне 2 часа удовольствия.
Кино

У Шварцнегера это большое у Д.Чана маленькое у Мадонны совсем нет,
а папа Римский давно этим не пользуется
Фамилия

Слово из трех букв, которого боится любой мужчина?
Еще!

Какая разница между пивом и лекарством?
Лекарство сначала выписывают, а потом пьют, а с пивом наоборот

Это блюдо атрибут В праздник блюдо подают.
Он на праздничном столе, А зовется:
Оливье

День рождения — хорошо, Завтра будет же бо-бо,
После водки и вина. Ох, болит же голова.
Похмелье

Что скрывает именинник, Пряча в темный холодильник?
Это вкусно и свежо, Наливай же нам
Вино

Не поможет вам химчистка, Если зубы растут близко.
А нужна вам лишь родная, Деревянная такая. ..
Зубочистка

Что у женщины на теле, У еврея на уме,
Применяется в хоккее И на шахматной доске?
Комбинация

Чем отличается женщина от телевизора?
Телевизор сначала показывает, а потом ломается

Сначала маленький, вяленький; потом упругий и большой!
Воздушный шарик

Какая разница между директором и мужем?
Директор знает своего заместителя, а муж нет

Отгадай загадку,что под яйцами гладко?
Сковородка

Беру двумя руками,сую между ногами, пять минут потею,а потом балдею???
Велосипед

С когтями, а не птица, летит и матерится.

Электромонтер

Если б не бабушкины лохматушки — мерзли бы дедушкины колотушки.

Хорошие загадки приятно загадывать и интересно отгадывать. Это развивает мышление, логику и расширяет кругозор. Вся семья может интересно провести вечер с друзьями, если устроить викторину.

Интересные загадки с подвохом с ответами

Когда в вопросе скрывается подтекст, не различимый сразу, он становится вдвойне сложным. Пускаясь в рассуждение относительно вопроса, порой забывают про условие. А тут важна каждая мелочь. Иначе ответ получится неверным. По этому принципу строятся сложнейшие загадки с множеством данных и цифр, но ответ лежит на поверхности, и даже ребенок может дать правильный ответ.

Интересные загадки с ответами:

  • Ты — пилот самолета, который летит по маршруту «Париж — Амстердам» с двумя пересадками в Берлине. Как фамилия летчика?

/В условии сказано, что ведешь самолет ты. Значит, и фамилия тоже твоя./

  • Во всем доме отключили свет. Но в чулане находится газовая плита, там же есть керосиновая лампа и свечка. В спичечном коробке осталась только одна спичка. Что ты зажжешь в первую очередь?

/Сначала следует зажечь спичку./

  • По проселочной дороге в сильный ливень ехал автобус. Все пассажиры уснули, только один водитель бодрствовал. Как его звали и какой номер маршрута?

/В такой ливень не различимы номера. А водитель — Анатолий. /

  • Проводится экзамен в военном училище связи. В аудитории тишина, лишь изредка преподаватель стучит карандашом по столу. Курсант берет билет и садится за стол. Через минуту он молча подходит с зачеткой, и преподаватель, не задав ни одного вопроса, ставит ему «отлично». Счастливый студент уходит. Почему?

/Преподаватель проверял и знания, и бдительность студентов. Для этого он, используя азбуку Морзе, выстукивал: «Кто подойдет сейчас ко мне, получит «отлично». Студент один из всех отреагировал правильно./

Иногда загадка похожа на маленькую задачку: есть условие, есть вопрос. Для малышей, пожалуй, это будет сложновато. Необходимо держать в памяти условие, чтобы отгадывать. И кругозор еще невелик. Но для деток младшего школьного возраста интересные загадки на логику с ответами будут полезны:

  • Из какой чашки невозможно напиться?

/Из пустой./

  • Под каким кустарником спрячется во время дождя заяц?

/Все кусты и деревья во время дождя мокрые. Заяц укроется под мокрым кустом./

  • Два абсолютно одинаковых человека идут навстречу друг другу. Кто из них поприветствует другого первым?

/Тот, кто более вежлив./

  • Вес полной бочки воды — 100 килограмм. Что следует сделать, чтобы ее вес изменился на 10 килограмм?

/Проделать в ней дырку./

  • Что сдерживают, не трогая руками?

/Это — собственное дыхание./

  • Электропоезд движется в направлении по ветру. Куда направлен дым?

/Он не имеет дыма, топки в других конструкциях поездов./

  • Известно, что белые медведи — хищники. Тем не менее нерпу они едят, а пингвина — нет. Почему?

/Просто они не встречаются, пингвины не живут на Севере./

  • Когда небо бывает ниже земли?

/Когда оно отражается в луже./

  • Какой бы огромной ни была кастрюля, есть кое-что, что никогда в нее не поместится. Что это?

/Крышку кастрюли всегда делают чуть большей окружности. /

  • Это находится в комнате, но в нем — сама комната. Что же это?

/Зеркало./

Короткие логические загадки

Так называются задачи, в которых требуется составить логическую последовательность рассуждений. Она и приведет к ответу. Здесь тоже внимательность к условию — залог правильного решения. Вот интересные логические загадки с ответами:

  • Человеку нельзя спать в течение семи дней. Тем не менее он нашел выход. Какой?

/Он решил, что будет спать по ночам — ведь нельзя это делать только днем./

  • Один фермер возвращался домой с поля. Вдруг налетел сильный ветер и начался ливень. Фермеру нечем было защититься, и домой он пришел совершенно мокрый. Но выяснилось, что ни один волос у него на голове не промок! Как он этого добился?

/Фермер был лысым./

  • Один преподаватель физики наглядно демонстрировал законы природы. Он мог ненадолго установить закрытую банку на столе таким образом, что большая часть ее нависала над полом. Правда, потом банка все же падала. В чем секрет физика?

/Он сначала замораживал в банке воду, положив ее горизонтально. Воды было приблизительно одна треть банки. Когда он ставил ее на стол вертикально, столбик льда всегда располагался на столе, а пустая часть — над полом. Когда лед начинал таять, вес распределялся по всему дну и банка падала./

Смешные загадки

Мозговой штурм может утомить. Поэтому чередуем сложные задачи со смешными вопросами. Главное в них — улыбка слушателя. Интересные загадки с ответами:

  • Есть слово, которое всегда произносят неверно. Что это за слово?

/Это слово «неверно»./

  • У него два рога, но это не бык, у него есть ноги, но нет копыт, он летит — громко воет, приземлится — землю роет.

А вот цепочка веселых загадок, которая развеселит и детей, и взрослых:

    • Как на раз-два-три поместить жирафа в холодильник?
      /1. Открываем. 2. Суем туда жирафа. 3. Закрываем./
    • Как на раз-два-три-четыре поместить слона в холодильник?
      /1. Открываем. 2. Выбрасываем жирафа. 3. Суем слона. 4. Закрываем./
    • Лев приказал всем явиться на собрание. Один не пришел. Кто? /Это слон, он сидит в холодильнике./
    • Как перебраться через реку, полную крокодилов? /Быстро, пока крокодилы не вернулись с собрания./

Для взрослых

Некоторые крупные коммерческие корпорации при отборе сотрудников используют логические задачи. Когда нужен не интеллект, а способность принимать нестандартные решения. Эти вопросы, над которыми предстоит хорошенько подумать, придутся по вкусу родителям и старшим школьникам. Здесь есть над чем поломать голову. Интересные загадки для взрослых с ответами:

  • Узнику дают шанс освобождения при условии, что он правильно решит свою участь. В противном случае его ждет казнь. Его ставят перед двумя дверями, которые охраняют два стража. Известно, что один всегда говорит правду, а другой — через раз. Узник имеет право задать два вопроса только одному, но он не знает, кто есть кто из стражей. Он смог решить задачу и освободился. Как он это сделал?

/1. Ты хитрец? 2. Эта дверь на свободу?/

  • Мистер Джонс позвонил в полицию и сообщил, что его грозится убить мистер Уиллс. Он настаивал, чтобы к нему немедленно выслали кого-нибудь. Прибывший на место детектив обнаружил тело Джонса. Включив диктофон Джонса, он услышал: «Уиллс убьет меня. Он уже тут. Скрываться бесполезно. Эта запись станет основанием для его ареста». Детектив понял, что это ложный след. Истинный преступник был обнаружен позднее. Почему не арестовали Уиллса?

/Перемотанная пленка диктофона говорила о сфабрикованности улики против Уиллса. Если он видел, как Джонс что-то говорит в диктофон, он бы уничтожил запись./

Для детей

  • По чему ты идешь спать на кровать?

/По полу./

  • Что произойдет с красным платком, если его опустить в речку?

/Намокнет./

  • Напиши слово «мышеловка» с помощью пяти букв.
  • Что у овцы — одна, у воробья — две, а у мышки — ни одной?

/Буква «о». /

  • Назовите полотно, из которого не получится ничего сшить.

/Железнодорожное./

  • Назовите леса, в которых никто не живет.

/Строительные./

  • Назовите колесо автомобиля, которое во время движения не вращается.

/Запасное./

  • Назовите пять дней подряд, не называя при этом чисел.

/Послезавтра, завтра, сегодня, вчера, позавчера./

  • Назовите первую женщину-летчика.

/Баба-Яга./

  • Какой гребень не годится для причесок?

/Петушиный./

  • Как носят воду в решете?

/В виде льда./

Для всей семьи

Рассказы про Шерлока Холмса с интересом читают и взрослые, и дети. Сразу хочется применить его дедуктивный метод, а потом сказать лучшему другу: «Элементарно, Ватсон!» А если бы в его время были сотовые телефоны, это было бы совсем просто. Для вас — интересные загадки с ответами, где фигурирует знаменитый детектив.

  • Как-то раз, прогуливаясь по Лондону, Шерлок Холмс заметил лежавшую на земле женщину. К сожалению, она была мертва. Холмс достал из ее сумочки телефон и нашел там номер супруга. Позвонив ему, он сообщил: «Ваша жена умерла. Срочно приезжайте». Когда тот прибыл, его уже ждала полиция. Холмс указал на него, как на преступника. Почему?

/Холмс не назвал улицу, где нашел труп./

А вот задачка про жителей отдаленной российской глубинки:

  • Жила на хуторе девочка-сирота. У нее были 3 кошки, 3 собаки, 3 попугайчика и 2 хомячка. У хомячков родились детки, 7 штук. Понадобилось купить корм этим животным. В магазин дорога шла сначала полем, потом лесом, потом снова полем, потом снова лесом, потом снова полем. Но оказалось, что корм не завезли. Пришлось ей идти в другой магазин: полем, лесом, полем, лесом, полем, лесом. Это был незнакомый лес, и девочка свалилась в хорошо замаскированную яму. Тут она услышала, что ей ставят условие: если она выберется из ямы, умрет ее мама. Если останется в яме, умрет ее папа. Как ей быть?

/Выбираться из ямы, потому что у нее давно все умерли. /

Просмотрев все эти интересные загадки с ответами, можно вывести принципы, по которым их составляют, и попробовать порадовать близких новыми вариантами про Шерлока Холмса или запутанными задачками, в которых главное — хорошо помнить условие.

И маленькие детишки, и ребята школьного возраста безумно любят совместные игры с родителями, бабушками или дедушками. Поэтому интересные загадки с ответами обязательно привлекут их внимание. Самое главное, чтобы взрослые продумали сценарий, по которому будет проходить увлекательная игра.

Загадка как способ развития ребенка

Вообще, интересные с ответами — не просто вдохновляющая игра. Это способ в развлекательной форме развить:

  • мышление;
  • логику;
  • фантазию;
  • усидчивость;
  • стремление.

Это лишь некоторые факторы, указывающие на то, что сложные интересные загадки с ответами — это не только весело, но и полезно для чада.

Увлекательная игра с логическим уклоном

Конечно же, лучше всего перевести задания в игровую форму. Это можно сделать с учетом того:

  • сколько детей принимает участие в мероприятии;
  • какого возраста ребята;
  • в чем задача игры.

Можно эстафету, в которой каждый ребенок сможет проявить смекалку и скорость мышления. Интереснее будет, если деткам вручать монетки за каждый правильный ответ. Потом, в конце игры, можно монетки обменять на какую-то сладость или игрушку. В игровой форме детки не будут воспринимать задание как урок, поэтому его будет выполнять гораздо интереснее и веселее.

Самые интересные загадки с ответами на логику

Задачки на мышление помогут проверить, насколько ребенок умеет мыслить нестандартно. Именно для этой цели понадобятся интересные загадки с ответами.

В комнате три дивана, у каждого из них по четыре ножки. Еще в комнате пять собак, у каждой из них по четыре лапы. Позже зашел в комнату человек. Сколько ног в комнате стало?

(Две, у дивана — не ноги, а у животных — лапы.)

Меня зовут Витя, мою младшую сестру — Алена, среднюю — Ира, а старшую — Катя. Как зовут брата каждой из сестер?

Какое колесо автомобиля не шевелится во время правого поворота?

(Запасное.)

Где оказался великий путешественник Геннадий, когда свеча в его руках потухла?

(В темноте.)

Ходят, а с места ни на шаг.

Два друга играли в футбол три часа. По сколько времени играл каждый из них?

(По три часа.)

Как называется слон, у которого нет хобота?

(Шахматный.)

Шла девочка Арина в сторону дачи и несла в корзине яблочные пирожки. Навстречу шли Петя, Гриша, Тимофей и Семен. Сколько всего детей шло на дачу?

(Только Арина.)

Что постоянно становится больше, а меньше никогда?

(Возраст.)

Бабушка несла продавать двести куриных яиц. По пути у нее оторвалось дно пакета. Сколько яиц она донесет до рынка?

(Ни одного, из порвавшегося дна все выпали.)

Логические интересные загадки с ответами понравятся детям. Взрослые также с огромным удовольствием будут обдумывать такие вопросы.

Увлекательные и интересные загадки с хитрым ответом

Также стоит обратить внимание на задания, разгадки в которых совершенно непредсказуемы. Интересные загадки с ответами представлены вашему вниманию ниже.

Какой станет зеленая футболка, если в ней зайти в Черное море?

Животное, которое есть в зоопарке, а также на пешеходной зоне трассы.

Горят два дома. Один дом богатых людей, а другой — бедных. Какой из домов первым потушит скорая помощь?

(Скорая помощь не тушит пожары.)

Сколько лет в году?

(Одно лето.)

Его можно завязать, но нельзя развязать.

(Разговор.)

Перед кем даже короли и лорды снимают шляпы?

(Парикмахер.)

В вагоне метро ехало пятнадцать человек. На одной остановке вышло трое, а зашло пятеро. На следующей остановке никто не выходил, а зашло три человека. На еще одной остановке вышло десять человек, а зашло пятеро. Еще на одной остановке вышло семь человек, зашло трое. Сколько всего остановок было?

Река, которая есть даже во рту у человека.

Муж жене подарил колечко и сказал: «Я уезжаю работать за границу. Когда я уеду, посмотри, что написано на внутренней стороне украшения». Когда жене было весело, она читала надпись, и ей становилось грустно, а когда ей было грустно, то надпись дарила силы. Что было написано на кольце?

(Все пройдет.)

Что ты можешь взять в левую руку, но никогда не сможешь этого взять правой рукой?

(Правый локоть.)

Вот такие интересные загадки с ответами помогут малышу пошевелить извилинами и хорошенько подумать.

Загадки на логику для самых маленьких

Самым маленьким детишкам лучше всего предлагать к разгадыванию самые несложные для восприятия задачки.

В саду на елке выросло пять яблок, а на березе — четыре груши. Сколько всего плодов?

(Нисколько, на этих деревьях фрукты не растут.)

Из какой тарелки ты ничего не сможешь скушать?

(Из пустой.)

В вазочке четыре ромашки, три розы, два тюльпана и две хризантемы. Сколько всего ромашек в вазе?

(Четыре ромашки.)

Витя сделал три горочки песка. Потом он их все соединил в одну и добавил еще одну собранную горочку. Сколько всего горок получилось?

Пришел декабрь, в саду у бабушки поспели вишни и малинка. Сколько деревьев или кустов подарили плоды?

(Ни одного, в декабре плоды не растут.)

Две сестры-близняшки Аня и Таня решили устроить игру и во время отпуска договорились, что одна будет говорить только правду, а вторая всегда будет говорить неправду. Девочки со двора придумали, как вычислить, кто из них врет. Какой вопрос они задали?

(А солнце светит?)

В снегу она одна, в морозе ее нет, а в сосиске их целых три. Что это?

(Буква «С».)

Какой человек не намочит волосы даже под ливнем?

Может ли павлин сказать, что он птица?

(Нет, потому что павлины не разговаривают.)

Два мальчика залезли на чердак, чтобы найти там старые игрушки. Когда они вышли на солнечный свет, было видно, что у одного лицо все запачканное, а у второго чистое. Умываться первым пошел тот мальчик, лицо которого чистое. Почему?

(Он увидел, что второй грязный, и подумал, что тоже такой.)

Сколько йогуртов ты можешь скушать на пустой желудок?

(Один, остальные не на голодный желудок. )

Насколько быстро должна бежать кошка, чтобы банка, привязанная к хвосту, не шумела?

(Кошка должна сидеть на месте.)

Логические загадки для школьников

Мальчишкам и девчонкам, которые посещают школу, стоит задавать задачки посложнее, где нужно хорошенько подумать. Давайте рассмотрим, какие детские интересные загадки с ответами можно включить в развлекательное мероприятие.

Как можно прыгнуть с двадцатиметровой лестницы и не удариться?

(Прыгнуть с нижних ступенек.)

У собаки на шее была двенадцатиметровая цепь. Она прошла более двухсот метров. Как так получилось?

(Она не была привязана.)

Что делать, если ты видишь зеленого человечка?

(Переходить пешеходный переход.)

Может ли в комнате человек находиться без головы?

(Да, если высунул голову в форточку или окно.)

Можно ли увидеть прошлогодний снег? Когда?

Когда белой кошке будет удобнее попасть в темную комнату?

(Когда открыта дверь.)

У тебя в руках спичка, в темной комнате при входе стоит свеча и плита. Что зажжешь в первую очередь?

Что весит больше — один килограмм сладкой ваты или один килограмм железных гвоздей?

(Весят одинаково.)

Сколько крупинок гречки войдет в стакан?

(Нисколько, крупинки не ходят.)

У каждой из четырех родных сестер Анжелы, Кристины, Ольги и Ирины есть по одному брату. Сколько в семье детей?

Пришла в больницу на обследование. Она была сестрой доктора, но доктор не был ее братом. Кем был врач?

(Сестрой.)

Настя и Алиса играли игрушками. Одна из девочек играла с плюшевым мишкой, а другая — с машинкой. Настя не играла с машинкой. Какая игрушка была у каждой девочки?

(Настя — с мишкой, а Алиса — с машинкой.)

Сколько углов станет у прямоугольного стола, если отпилить один угол?

(Пять углов.)

Настя и Кристина вдвоем пробежали восемь километров. По сколько километров пробежала каждая девочка?

(По восемь каждая.)

Эти очень интересные загадки с ответами помогут ребенку показать и умственные способности. Родителям стоит проявить фантазию и устроить настоящий марафон эмоций.

Почему следует загадывать загадки

Совместное времяпровождение очень нужно малышу, чтобы он понимал, как родители его любят. Поэтому следует чаще устраивать такие мероприятия. Ребенок также сможет проявить свои таланты в процессе игры.

Веселый праздник

Мамам, папам, бабушкам и дедушкам следует понимать, что чем ярче будет событие, тем интереснее и веселее будет ребенку. Поэтому стоит:

  • устроить карнавал, в котором все будут в красивых костюмах;
  • придумать подарки для победителя эстафеты;
  • награждать того, кто набрал максимум очков, какими-то презентами за каждый правильный ответ.

Детишки будут рады любому событию. А когда обычный вечер превращается в праздник, то счастья не будет предела. Все зависит от фантазии и идей родителей. Радуйте своих маленьких сыночков и доченек, и они отблагодарят вас блеском в глазках и довольными улыбками.

⚡ Пошлые загадки ⚡

Что объединяет горелый хлеб, утопленника и беременную женщину?
ОТВЕТ: Не успели вытащить. ..

У какого молодца утром капает с конца?
ОТВЕТ: Самовар.

Кто такой: Сам не стреляет и другим не дает?
ОТВЕТ: Александр Матросов.

В темной комнате, на белой простыне — два часа удовольствия.
ОТВЕТ: Киносеанс.

Что у Адама спереди, а у Евы сзади?
ОТВЕТ: Буква «А».

Туда-сюда-обратно: Тебе и мне — приятно.
ОТВЕТ: Качели.

Мальчик с девочкой в траве что-то делали на «Е».
ОТВЕТ: Ели землянику.

Около 40 млн. человек занимаются ЭТИМ по ночам. Что ЭТО такое?
ОТВЕТ: Internet.

Стоит баба на полу, приоткрыв свою дыру.
ОТВЕТ: Печка.

В зубах доска, в глазах тоска.
ОТВЕТ: Мужик провалился в деревенский толчок.

Без рук, без ног на бабу скок!
ОТВЕТ: Коромысло.

Прыгает ловко и ест морковку?
ОТВЕТ: Бубка на диете.

Не петух, а поет, не дед, а бабку имеет, кто это?
ОТВЕТ: Филипп Киркоров.

Одно колесо тысяча крыльев — что это?
ОТВЕТ: Тачка с навозом.

Что такое: твердое в мягкое вставляется, и шарики рядом болтаются?
ОТВЕТ: Сеpьги.

Что возбуждается палочкой Коха?
ОТВЕТ: 1. туберкулез; 2. жена Коха.

Как можно ходить сидя?
ОТВЕТ: В туалете — на унитазе.

Морщинистый Тит всю деревню веселит.
ОТВЕТ: Нехватка молодежи на селе.

В две руки беру, между ног сую, пять минут потею, а потом балдею.
ОТВЕТ: Велотренажер.

Волосатая головка за щеку летает ловко.
ОТВЕТ: Зубная щетка.

С когтями, а не птица, летит и матерится.
ОТВЕТ: Электромонтер.

То висячий то стоячий то холодный то горячий.
ОТВЕТ: Душ.

Четыре братца под одной крышей стоят.
ОТВЕТ: Мафия.

Красная головка — работает ловко.
ОТВЕТ: Дятел.

Волоса, волоса…, а посредине колбаса.
ОТВЕТ: Кукуруза.

Маленькая сморщенная, есть в каждой женщине.
ОТВЕТ: Изюминка.

С луком с яйцами, но не пирожок?
ОТВЕТ: Робин Гуд.

Слово из трех букв, которого боится любой мужчина?
ОТВЕТ: Еще!

Между ног болтается, воняет и орет?
ОТВЕТ: Мотоцикл.

В небе одна, В земле нету, А у бабы их целых две.
ОТВЕТ: Буква Б.

Тихо сзади подошел, Дважды всунул и пошел.
ОТВЕТ: Тапочки.

Волос на волос, тело на тело — начинается темное дело.
ОТВЕТ: Глаз закрывается.

Зерно, прошедшее огонь, воду и медные трубы.
ОТВЕТ: Самогон

Новое название любимого музыкального инструмента американского президента.
ОТВЕТ: Кексофон

Что общего у водолаза и повара?
ОТВЕТ: И тому, и другому время от времени приходится опускать яйца в воду.

Что общего между деньгами и гробом?
ОТВЕТ: И то и другое сначала заколачивают, а потом спускают.

Шуточные загадки на Новый год 2022 с ответами для взрослых

Чтобы в 2022 году новогодний праздник прошел весело и запомнился всем на нём присутствующим, необходимо обязательно продумать и составить оригинальную развлекательную программу.

Заранее придуманные конкурсы и игры, проводимые во время этого мероприятия, никому не дадут заскучать и поднимут собравшимся людям настроение.

Многие из ведущих корпоративных вечеров предлагают участникам разнообразные загадки для взрослых на 2022 Новый год, которые можно отгадывать, сидя прямо за столом.

Загадки должны быть обязательно шуточными, но не злыми, и, конечно же, иметь отношение к новогоднему празднику, его главным персонажам или символу наступающего года.

Для незабываемой встречи 2022 года Черного Тигра отлично подойдут, например, следующие задания и загадки:

Знают даже дети –
Трезвой на банкете
Остается только
Праздничная … (ёлка)

Любой сотрудник в Новый год
Непременно чуда ждет,
И мечтает он – вот-вот
Ее начальник принесет! (премия)

Что на нашем празднике
Бегало, скакало?
А под бой курантов
Под елочкой упало? (Дед Мороз)

Дождь цветной кругом идет,
У гостей душа поет!
А хозяева стоят,
И тихонечко грустят.
Знают – завтра им мести
Круглешочки – … (конфетти)

Первый гость он на столе,
В каждом доме и семье,
Слегка уставшие месье
Отдыхают в … (оливье)

Не ружье, но стреляет,
Не змея, но шипит,
Не водка, но … (шампанское)

По всей стране куранты бьют,
Запускается салют!
Первый сразу же потух,
Второй совсем к поджогу глух,
Третий улетел к соседям,
Четвертый – в зоопарк к медведям,
Задача наша непроста –
CHINA – … (пиротехника)

Ежегодно в Новый год,
Утром на рассвете,
Кого приносит Дед Мороз?
Догадайтесь, дети! (папу с новогоднего корпоратива)

Новый год – чудесный праздник,
Но без нее не будет счастлив!
Купить ее необходимо,
Иначе гости пройдут просто мимо! (водка)

Какими бы ни были выбранные вами для игр и развлечений на Новый 2022 год загадки для взрослых, они точно разнообразят мероприятие и придадут ему особенное веселое настроение.

Считается, что как человек проведет новогоднюю ночь, таким окажется и весь его наступивший год. Поэтому необходимо хорошо постараться и сделать все возможное, чтобы торжество получилось увлекательным, веселым, оригинальным, понравилось и надолго запомнилось всем его участникам.

загадок для взрослых | 30 загадок с ответами

Кто сказал, что детям должно быть весело? Наши загадки для взрослых заставят вас отгадывать и хихикать!

Загадки и головоломки — это не только забавные головоломки, они отлично работают как ледоколы и помогают разрушить неловкое молчание. Посмотрим правде в глаза, независимо от вашего возраста, это простой способ весело провести время с друзьями, семьей или даже виртуальными незнакомцами. Они помогают людям узнать друг друга. От причудливых до немного дерзких, мы собрали коллекцию из 30 загадок для взрослых, которым нужен вызов и смех.


Веселись с друзьями

Загадки — отличный способ скоротать время, но это не единственное, что предлагает Let’s Roam! Наши виртуальные игровые вечера позволяют взрослым общаться с 15 любимыми людьми через очень простой в использовании интерфейс. Выбирайте из более чем 15 вариантов и сделайте игровые вечера обычным явлением! Наши профессионалы собрали самые лучшие викторины, игры для рисования, декодеры смайликов и многое другое. Они подходят для семейного отдыха, но их также можно использовать для виртуальных счастливых часов или тимбилдинга! Кроме того, любое из наших развлечений для публики может быть настроено индивидуально, поэтому не стесняйтесь обращаться к нашим специалистам.


1. Загадка: Прекрасная и круглая, я мерцающе-белая, выросшая во тьме, женское наслаждение. Что я?
Ответ: Жемчужина

2. Загадка: Две мамы и две дочери пошли обедать и каждая съела по одной порции. Всего было съедено всего три порции, как такое возможно?
Ответ: Это были бабушка (которая является матерью), мать (которая является матерью и дочерью) и дочь, которые вышли на ужин.

3. Загадка: Что разбивается, но никогда не падает, и что падает, но никогда не разбивается?
Ответ: День и ночь

4. Загадка: Люди месят меня, чтобы купить вещи. Что я?
Ответ: Тесто

5. Загадка: Вы входите в комнату с кроликом, держащим морковку, свиньей, поедающей помои, и шимпанзе, держащим банан. Какое животное в комнате самое умное?
Ответ: Надеюсь, вы.

6.Загадка: Если 5 эльфам нужно 5 минут, чтобы сделать 5 кукол, то сколько времени потребуется 100 эльфам, чтобы сделать 100 кукол?
Ответ: 1 эльфу требуется 5 минут, чтобы сделать куклу, поэтому 100 эльфам потребуется 5 минут, чтобы сделать 100 кукол.

7 . Загадка: Человек застрял в комнате. Из комнаты есть только два возможных выхода: две двери. За первой дверью находится комната, построенная из увеличительного стекла. Палящее горячее солнце мгновенно поджаривает все и всех, кто входит.Во второй двери есть огнедышащий дракон. Как убегает мужчина?
Ответ: Он ждет до ночи, а затем проходит через первую дверь.

8. Загадка: Определенное преступление наказуемо при покушении, но не наказуемо при совершении. Что это?
Ответ: Самоубийство

9. Загадка: Мистер Браун был убит в воскресенье днем. Его жена сказала, что в то время она читала книгу. Дворецкий сказал, что принимает душ.Шеф-повар сказал, что готовит завтрак. Горничная сказала, что складывает одежду, а садовник сказал, что сажает помидоры. Кто сделал это?
Ответ: Шеф-повар. Мистер Браун был убит днем, а шеф-повар утверждал, что готовит завтрак?

10. Загадка: Если у вас есть трехлитровый кувшин, пятилитровый кувшин и неограниченный запас глинтвейна, как вы без подсчетов получите ровно четыре литра?
Ответ: Наполните пятигаллонный кувшин. Затем наполните трехгаллонный кувшин из пятигаллонного, оставив два галлона.Вылейте вино из трехгаллонного кувшина. Перелейте вино из пятигаллонного кувшина в трехгаллонный. Наполните пятигаллонный кувшин и используйте его, чтобы заполнить оставшееся место на один галлон в трехгаллонном кувшине. У вас останется четыре галлона вина в пятигаллонном кувшине.

Ищете более простые загадки для взрослых? Мы вас прикрыли!

11. Загадка: Сколько бананов можно съесть, если желудок пуст?
Ответ: Всего один.После этого ваш желудок уже не пуст.

12. Загадка: Я могу быть горячим или холодным. Я могу бежать и быть неподвижным. Я могу быть твердым и мягким. Что я?
Ответ: Вода

13. Загадка: Как узнать, что Юпитер, Сатурн, Уран и Нептун собираются пожениться?
Ответ: У всех есть кольца.

14. Загадка: Как дети могут пить пиво и не напиться?
Ответ: Они могут пить корневое пиво!

15.Загадка: Мужчина отправляется в круиз между Мексикой и США. Он ни с кем не разговаривает, не останавливается ни в одном порту и даже не выходит из своей каюты, но зарабатывает на своем путешествии 300 000 долларов. Как?
Ответ: Он контрабандист.

16. Загадка: Женщина стреляет в своего мужа. Затем она держит его под водой более 5 минут. Наконец, она вешает его. Но уже через 5 минут они вдвоем выходят на улицу и наслаждаются прекрасным ужином. Как это может быть?
Ответ: Женщина была фотографом.Она сделала снимок мужа, проявила его и повесила сушиться.

17. Загадка: Мужчина приехал в город во вторник и уехал через два дня, во вторник. Как же так?
Ответ: Вторник так звали его лошадь.

18. Загадка: Почему нельзя сфотографировать человека с деревянной ногой?
Ответ: Деревянная ножка не фотоаппарат.

19. Загадка: У меня четыре крыла, но я не могу летать. Я никогда не смеюсь и никогда не плачу.Меня всегда находят на одном и том же месте, я беззвучно тружусь. Что я?
Ответ: Ветряная мельница!

20. Загадка: Вы видите лодку, полную людей, но на борту нет ни одного человека. Как это возможно?
Ответ: Все женаты.

Думаете, эти загадки слишком просты для взрослых? Попробуйте свои силы в действительно сложных загадках!

21. Загадка: Меня берут из шахты и запирают в деревянный ящик, из которого я никогда не выхожу; но меня используют почти все.Что я?
Ответ: Грифель карандаша.

22. Загадка: По улице шли мужчина с собакой. Человек ехал, но шел. Как звали собаку?
Ответ: Еще.

23. Загадка: Что делает собака, на которую наступает человек?
Ответ: Штаны.

24. Загадка: Что имеет четыре ноги, как у собаки, два глаза, как у собаки, выглядит так же, как собака, но не является собакой?
Ответ: Изображение собаки.

25. Загадка: У кого есть шея, но нет головы?
Ответ: Бутылка.

26. Загадка: У кого есть голова, но нет шеи?
Ответ: Пиво.

27. Загадка: Чем отличается тюремщик от ювелира?
Ответ: Тюремщик следит за камерами, а ювелир продает часы.

28. Загадка: Как нарисовать линию, а затем удлинить ее, не касаясь ее?
Ответ: Мне пришлось бы провести более короткую линию.Тогда первая строка будет длиннее.

29. Загадка: Если бы все в США купили белую машину, какой была бы страна?
Ответ: Белая автомобильная нация.

30. Загадка: У меня есть ключи, но нет замков. Вы можете войти, но вы не можете выйти наружу. У меня есть место, но нет комнаты. Что я?
Ответ: Клавиатура.

Знаете ли вы, что загадки помогают нам понять человеческий разум?


Не позволяйте смеху остановиться на этом!

На этом наш список загадок для взрослых заканчивается, но приключения только начинаются.Теперь, когда у вас есть классные загадки, как вы будете их использовать? Вы все еще пытаетесь понять один? Дайте нам знать об этом в комментариях!

Кстати, если вы любите загадки, вам может понравиться наша охота за мусором! Вам даже не нужно выходить из дома, чтобы попробовать его. Мы предлагаем охоту за мусором на дому для семей и пар. В этих приключениях под руководством приложения вы будете расшифровывать подсказки, отвечать на вопросы и делать забавные фотографии по пути. Вы даже можете настроить охоту за мусором, добавив шутки или поручив нашим планировщикам создать шутку специально для особого события, например, дня рождения.


Часто задаваемые вопросы

Где найти загадки для взрослых?

Мы отобрали 30 веселых загадок, от которых взрослые почесают в затылке. Если вы хотите повеселиться, ознакомьтесь с этим списком из более чем 150 загадок или получите бесплатную пробную версию виртуальных игровых ночей от Let’s Roam.

Взрослые по-прежнему ценят загадки?

Да! Загадки не только для детей. Загадки для взрослых отлично снимают стресс. Они могут сделать вас душой вечеринки или помочь вам познакомиться с кем-то.Проверьте больше загадок и убедитесь сами!

Где найти дурацкие загадки?

Этот список загадок для взрослых содержит хорошее сочетание забавных и каверзных загадок с сыром. Если вы не нашли то, что искали, попробуйте этот мега-список из более чем 150 загадок!

66 головоломок (с ответами) для взрослых, детей и всех, кто находится между ними

Отгадай загадку: почему дети любят головоломки? Ответ. Они предлагают часы бесплатных развлечений, которые заставляют нас мыслить нестандартно.Они также обеспечивают здоровую дозу смирения, когда наконец говорят (такой очевидный!) ответ. Но самые веселые головоломки — это когда вы — родитель — задаете их. Загадки и головоломки позволяют почувствовать себя намного умнее, чем вы есть на самом деле. Пока ваш ребенок пытается придумать ответ, его мозги будут трещать от энергии и сообразительности, что, как мы все можем согласиться, является отличным способом скоротать время.

Вот список легко запоминающихся головоломок, которые развлекут ваших детей и раз и навсегда докажут, что у вас есть больше ответов, чем у ваших детей.

  1. Какое слово начинается и заканчивается на Е, но состоит только из одной буквы?
    Конверт.
  2. Какой сыр изготавливается в обратном порядке?
    Эдам.
  3. Я высокий в молодости и низкий в старости. Что я?
    Свеча.
  4. В одноэтажном розовом доме жил розовый человек, розовый кот, розовая рыбка, розовый компьютер, розовый стул, розовый стол, розовый телефон, розовый душ — все было розовым! Какого цвета были лестницы?
    Лестницы не было, дом был одноэтажный.
  5. Кто больше: мистер Биггер, миссис Биггер или их ребенок?
    Малыш, потому что он немного больше.
  6. Майк мясник. Его рост 5 футов 10 дюймов. Что он весит?
    Мясо.
  7. Какие следующие три буквы в следующей последовательности? J, F, M, A, M, J, J, A, __, __, __
    S, O, N. Последовательность — первая буква месяцев года. Сентябрь, октябрь и ноябрь — следующие в последовательности.
  8. Перед горой.Был открыт Эверест, какая гора была самой высокой в ​​мире?
    Эверест. Он по-прежнему оставался самым высоким в мире. Просто его еще не обнаружили!
  9. Вы ведете автобус. На первой остановке вошли две женщины. На второй остановке заходят трое мужчин и выходит одна женщина. На третьей остановке заходят трое детей и их мама, а выходит мужчина. Автобус серый, а на улице дождь. Какого цвета волосы у водителя автобуса?
    Какого бы цвета ни были твои волосы! Помните, вы водитель автобуса!
  10. Представьте, что вы находитесь в лодке посреди моря.Внезапно вас окружают голодные акулы, готовые съесть вас. Как вы можете положить этому конец?
    Хватит воображать!
  11. Когда я у тебя есть, тебе сразу хочется поделиться мной. Но, если вы разделяете меня, у вас нет меня.
    Секрет.
  12. Что вы можете держать, даже не касаясь руками?
    Твое дыхание!
  13. Сначала я выбросил снаружи и приготовил внутри. Затем я съел снаружи и выбросил внутри.Что я ел?
    Кукуруза в початках.
  14. Лифт находится на первом этаже. В лифте четыре человека, включая меня. Когда лифт достигает первого этажа, один человек выходит и трое входят. Лифт поднимается на второй этаж, выходят два человека и входят шесть человек. Затем он поднимается на следующий этаж, никто не выходит, но входят 12 человек. На полпути к следующему этажу трос обрывается, и лифт падает на пол.Все остальные умирают в лифте. Как я выжил?
    Я выбрался на второй этаж!
  15. Мужчина сбежал из дома. Он убежал, потом повернул налево, пробежал такое же расстояние, снова повернул налево, пробежал такое же расстояние и снова повернул налево. Когда он вернулся домой, там было двое мужчин в масках. Кто они?
    Кетчер и судья.
  16. Что можно увидеть раз в минуту, два раза в минуту и ​​никогда в тысячу лет?
    Буква М.
  17. Какая шина не двигается, когда автомобиль поворачивает направо?
    Запасное колесо.
  18. Что становится влажнее, чем больше высыхает?
    Полотенце.
  19. Чем больше вы берете из него, тем больше он становится. Что это?
    Яма.
  20. Что может путешествовать по миру, оставаясь в одном углу?
    Штамп.
  21. Что находится в конце радуги?
    Буква Ш.
  22.  Сколько месяцев по 28 дней?
    Все 12 месяцев.
  23. Железнодорожный переезд, берегитесь автомобилей. Вы можете произнести это без Rs?
    Т-Х-А-Т.
  24. Две матери и две дочери пошли поесть. Все съели один бургер, но всего было съедено только три гамбургера. Как это возможно?
    Это были бабушка, мама и дочь.
  25. Мужчина гулял на улице, когда пошел дождь. У мужчины не было ни зонта, ни шляпы. Его одежда промокла, но ни один волос на голове не промок.Как такое могло произойти?
    Мужчина был лысым.
  26. Сколько секунд в году?
    Двенадцать. 2 января, 2 февраля, 2 марта…
  27. Вы идете по мосту и видите лодку, полную людей, но на борту нет ни одного человека. Как это возможно?
    Все люди на корабле женаты.
  28. У матери Бет три дочери. Одного зовут Лаура, другого Сара. Как зовут третью дочь?
    Бет.
  29. Что полно дыр, но все еще держит воду?
    Губка.
  30. Как вы можете бросать мяч так сильно, как только можете, только для того, чтобы он вернулся к вам, даже если он ни от чего не отскакивает?
    Подбросить мяч прямо вверх.
  31. У меня нечетное число. Убери одну букву и я стану квитком. Какой я номер?
    Семь. Если я уберу «с», оно станет «четным».
  32. Что никогда не задает вопросов, но на него часто отвечают?
    Звонок в дверь.
  33. Что принадлежит вам, но другие люди используют это больше, чем вы?
    Ваше имя.
  34. У меня есть большая копилка, 10 дюймов в ширину и 5 дюймов в высоту. Примерно сколько монет я могу положить в свою пустую копилку?
    Всего один, после чего он уже не будет пустым.
  35. Что это значит? I ПРАВО I
    Прямо между глаз.
  36. Какое пятибуквенное слово становится короче, если к нему добавить две буквы?
    Короткий.
  37. Что движется быстрее? Горячий или холодный?
    Горячий. Вы можете простудиться
  38. У матери Джимми было четверо детей. Она назвала первый понедельник. Она назвала второй вторник, а третью — среду. Как зовут четвертого ребенка?
    Джимми! У матери Джимми было четверо детей.
  39. Что особенного в словах: работа, полироль, трава?
    Все они произносятся по-разному, когда первая буква заглавная.
  40. Дайте мне еды, и я буду жить; дай мне воды, и я умру.Что я?
    Огонь.
  41. Что ходит на четырех ногах утром, на двух днем ​​и на трех вечером?
    Человек.
  42. У отца Питера пятеро сыновей. Имена четырех сыновей — Фефе, Фифи, Фафа и Фуфу соответственно. Как зовут пятого сына?
    Питер.
  43. Что летает без крыльев?
    Время.
  44. Переверни меня на бок, и я стану всем. Разрежь меня пополам, и я ничто. Что я?
    Число 8.
  45. Что есть города, но нет домов; леса, но нет деревьев; и вода, а рыбы нет?
    Карта.
  46. Что должно сломаться, прежде чем вы сможете его использовать?
    Яйцо.
  47. На какой вопрос ты никогда не сможешь ответить?
    «Ты еще не спишь?»
  48. Что можно сломать, даже не прикасаясь к нему?
    Обещание.
  49. Что поднимается вверх, но никогда не опускается?
    Ваш возраст.
  50. Я бреюсь каждый день, но мои волосы только отрастают.Что я?
    Парикмахер.
  51. Заходишь в комнату, а на столе спичка, лампа, свеча и камин. Что бы вы зажгли в первую очередь?
    Матч.
  52. Мужчина умер от старости в свой 25-й день рождения. Как?
    Он родился 29 февраля.
  53. Что не может говорить, но откликается, когда вы с ним говорите?
    Эхо.
  54. Чем больше этого, тем меньше ты видишь. Что это?
    Тьма.
  55. Я все время следую за тобой, но ты никогда не сможешь поймать или прикоснуться ко мне.Что я?
    Твоя тень.
  56. Что чистое, когда оно черное, и белое, когда оно грязное?
    Классная доска.
  57. Что имеет 88 ключей, но не может открыть ни один замок?
    Пианино.
  58. Что становится больше, чем больше вы забираете?
    Дырка.
  59. Когда сегодня предшествует вчера?
    В словаре.
  60. Слово, которое я знаю, содержит шесть букв, убери одну букву и останется 12. Что это?
    Десятки.
  61. Что всегда перед вами, но невидимо?
    Будущее.
  62. Можете ли вы добавить один математический символ между 55 555 и получить 500?
    555-55=500
  63. Какие буквы легко найти в любой день недели?
    ДЕНЬ.
  64. Наполни меня горячим или холодным. Засунь в меня что угодно, и я обязательно выдержу. Что я?
    Чашка.
  65. Какое второе имя у медведя Руперта?
    В.
  66. Что находится в центре Америки?
    Буква «р.

Ой! Пожалуйста, попробуйте еще раз.

Спасибо за подписку!

100 лучших загадок для взрослых с ответами [издание 2022 года]

Загадки — любимая игра всех времен, как для взрослых, так и для детей. Некоторые загадки могут быть забавными, а другие сложнее, чем вы думаете.Это языковые головоломки, в которых используется латеральное мышление.

Итак, если вы ищете хорошие, трудные и сложные загадки, чтобы обострить свой мозг, вы находитесь в правильном месте. Здесь вы найдете сложные загадки в основном для взрослых.

Не волнуйтесь, включены ответы.

 

100 супер хитрых загадок для взрослых

 

Загадки для взрослых. Часть 1

Загадка : Я блестящий драгоценный камень, я трансформировался, женское удовольствие.Что я?

Ответ: Алмаз

 

Загадка : Две матери и две дочери идут обедать, и каждая съела по одной порции. Всего съедено три порции, как это возможно?

Ответ: Это были бабушка, мать и дочь.

 

Загадка : Что разбивается, но никогда не падает, и что падает, но никогда не разбивается?

Ответ: Закат и рассвет

 

Загадка : Если 5 эльфам нужно 5 минут, чтобы сделать 5 игрушек, то сколько времени понадобится 100 эльфам, чтобы сделать 100 игрушек?

Ответ: 1 эльфу требуется 5 минут, чтобы сделать 1 игрушку, поэтому 100 эльфам может понадобиться 5 минут, чтобы сделать 100 игрушек.

 

Загадка: Я иду быстро и стою на месте. Я перевожу людей и вещи по всему миру. У меня есть места, но я не могу летать. Я издаю звуки, которые нравятся детям. Что я?

Ответ: Поезд.

 

Загадка : Я ухожу, когда тебе грустно или когда ты счастлив. В других случаях вы хотите спрятать меня, но это сложно, поэтому, пожалуйста, не пытайтесь. Что я?

Ответ: Слезы.

 

Загадка : Человек застрял в комнате.В комнате всего две двери, чтобы сбежать. За главной дверью есть область, построенная из увеличительного стекла. Жаркое солнце мгновенно поджаривает любого, кто входит. Через вторую дверь есть опасный дракон. Как мужчине спастись?

Ответ: Он ждет, пока солнце сядет, и входит в первую дверь.

 

Загадка : Что высокое, темное/серое и невидимое?

Ответ : Дым

 

Загадка : МистерБраун был убит в воскресенье днем. Его жена сказала, что в то время она читала газету. Дворецкий сказал, что принимает душ. Шеф-повар сказал, что завтракает. Горничная сказала, что мыла пол, а садовник сказал, что сажает салат. Кто сделал это?

Ответ: Повар. Мистер Браун был убит днем, а повар утверждал, что завтракал?

 

Загадка : У отца Алекса 3 дочери, Мари, Кристина, как зовут третью дочь?

Ответ: Алекс

 

Загадка : Когда Билли спрашивают, сколько ему лет, он отвечает: «Через два года я буду в два раза старше, чем был пять лет назад.» Сколько ему лет?

Ответ: Двенадцать лет

 

Загадка : Моя голова красная, но если ты меня поцарапаешь, она становится черной, кто я?

Ответ: Матч

 

Загадка : Что плохо пахнет, но вкусно?

Ответ: Язык.

 

Загадка: Мой гром предшествует молнии. Моя молния приходит раньше туч. Мой дождь сушит всю землю, которой касается. Что я?

Ответ: Вулкан

 

Загадка : Это принадлежит вам, но все остальные используют это

Ответ: Ваше имя

 

Загадка : Я вышел из шахты и окружен лесом.Я помогаю другим выражать себя. Что я?

Ответ: Грифель карандаша

 

Загадка : У меня есть города, но нет домов. У меня есть горы, но нет деревьев. У меня есть берега, но нет песка. Что я?

Ответ: Карта

 

Загадка : Ты считаешь мою жизнь часами, а я служу тебе, исчерпав их. Я быстрый, когда высокий, и медленный, когда низкий. Ветер — мой самый большой страх. Что я?

Ответ: Свеча

 

Загадка : У меня есть ключи, но нет замков.У меня есть место, но нет комнаты. Вы можете войти, но не можете выйти наружу. что я?

Ответ: Клавиатура компьютера

 

Загадка : Что имеет четыре пальца и большой палец, но не живое?

Ответ: Перчатка

 

Загадка : Когда я тебе нужен, ты меня бросаешь, когда я бесполезен, ты возвращаешь меня обратно, кто я?

Ответ: Якорь.

 

Загадка : Что исчезает, как только вы произносите его имя?

Ответ: Тишина

 

Загадка : Я говорю без рта и слышу без ушей.Я оживаю с ветром. Что я?

Ответ: Эхо

 

Загадка : Если я у тебя есть, ты разделишь меня. Если вы разделяете меня, у вас нет меня. Что я?

Ответ: Секрет

 

Загадка : У меня всего два слова, но тысячи букв. Что я?

Ответ: Почта

 

Загадка : Переверни меня на бок, и я стану всем. Разрежь меня пополам, и я ничто.Что я?

Ответ: число восемь

 

Загадка : Я действительно легкий, но меня очень трудно удержать, кто я?

Ответ: Дыхание

 

Загадка : Представь, ты в темной комнате, без окон и с запертой дверью, кто я?

Ответ: Просто перестаньте представлять себе комнату.

 

Загадка : Что может бегать, но никогда не ходить, иметь рот, но никогда не говорить, иметь голову, которая никогда не плачет, и постель, которая никогда не спит?

Ответ: Река.

 

Загадка : Сколько букв в «алфавите»?

Ответ: Слово «Алфавит» состоит из одиннадцати букв

 

Загадки для взрослых. Часть 2

Загадка : Как ты можешь бросать мяч так сильно, как только можешь, только для того, чтобы он вернулся к тебе, не отскакивая ни от чего?

Ответ: Подбросить мяч прямо в воздух.

 

Загадка : Что может заполнить всю комнату, не занимая при этом места?

Ответ: Свет.

 

Загадка : Это нужно всем, и все дают, но никто не берет, когда вы даете им. Что я?

Ответ: Совет.

 

Загадка : Что имеет корни, которых никто не видит и поднимается намного выше деревьев? Но не растет; что это?

Ответ: Гора.

 

Загадка : Два отца и два сына идут с рыбалки. Но когда они возвращаются домой, их остается только трое.Как это возможно?

Ответ : Они дедушка, отец и сын.

 

Загадка : Какое английское слово сохраняет эквивалентное произношение, даже если убрать четыре из пяти букв?

Ответ: Очередь.

 

Загадка : Убийца приговорен к смертной казни и ему предоставляется возможность умереть в одной из следующих трех комнат: комната, полная огня, комната, полная убийц с заряженным оружием, и область, заполненная львами, которые не съедено за годы.Какую комнату ему выбрать?

Ответ: Пространство со львами, потому что если они не ели годами, то они уже мертвы.

 

Загадка : Чем больше вы создаете, тем больше оставляете после себя. Что я?

Ответ : Следы

 

Загадка : Мужчина убегает из дома. Он поворачивает налево и продолжает бежать. Через некоторое время он снова поворачивает налево и продолжает бежать. Позже он в последний раз поворачивает налево и бежит домой.Кем был человек в маске?

Ответ: Ловец. В игре в бейсбол.

 

Загадка : Я охотник, но у меня нет ружья, я ловлю сети, но я не рыбак, кто я?

Ответ: Паук

 

Загадка : Когда ты слышишь обо мне, я заслуживаю всего, но когда ты узнаешь, кто я, я ничто. Кто я?

Ответ: Загадка.

 

Загадка : Что нельзя сжечь во время пожара и не тонуть в воде?

Ответ: Лед.

 

Загадка : Что находится посреди земли?

Ответ: Буква «Р».

 

Загадка : Я умею летать, но у меня даже нет крыльев. Я могу плакать, но у меня даже нет глаз. Куда бы я ни пошел, тьма следует за мной. Что я?

Ответ: Облако.

 

Загадка : Если вы участвуете в гонке и обгоняете человека, занявшего второе место, на каком месте вы находитесь?

Ответ: Второе место.

 

Загадка : Двое мужчин в пустыне.Они оба собрались. Один из мужчин мертв. У живого рюкзак открыт, у мертвого рюкзак закрыт. Что в рюкзаке?

Ответ: Парашют.

 

Загадка : Что может сделать ребенок такого, чего никто никогда не увидит?

Ответ: Шум.

 

Загадка : В большинстве случаев мой цвет красный, я красивая, меня все любят, особенно дамы. Иногда я могу причинить тебе боль. Я не это имел в виду, извините.Что я?

Ответ: Роза.

 

Загадка : У меня больше нет глаз, но когда-то я мог видеть. У меня есть рот, но я не могу говорить, теперь я белый и пустой. Что я?

Ответ: Череп.

 

Загадка : Что ты можешь держать в правой руке, но никогда в левой?

Ответ: Ваша левая рука.

 

Загадка : Все приходят и уходят, держа в руках?

Ответ: Дверь.

 

Загадка : Сколько месяцев по 28 дней?

Ответ: Все 12 месяцев

 

Загадка : Что приходит раз в минуту, дважды в мгновение, но никогда в тысячу лет?

Ответ: Буква «М»

 

Загадка : Когда ты меня хоронишь, я вырастаю.

Ответ: Семя.

 

Загадка : Я даю ответ «да», но я имею в виду «нет». Какой был вопрос?

Ответ: Вы не возражаете?

 

Загадка : Меня можно записать, меня можно произнести, меня можно раскрыть, меня можно сломать.Что я?

Ответ: Новости.

 

Загадка : У меня есть лучи, но я не солнце, я бегу, но у меня нет ног. Что я?

Ответ: Велосипед.

 

Загадка : Я все слышу, но ничего не могу сказать. Что я?

Ответ: Ухо

 

Загадка : Что имеет четыре ноги, но не может ходить?

Ответ: стол.

 

Загадка : Если ты видишь, то не видишь меня, а если не видишь, то видишь меня.Что я?

Ответ: Сон.

 

Загадки для взрослых. Часть 3

Загадка : Его нельзя увидеть, нельзя почувствовать, нельзя услышать и нельзя понюхать. Он лежит за звездами и под холмами, пустые ямы он заполняет. Кончает жизнь и убивает смех. Что это?

Ответ: тьма

 

Загадка : Что если у тебя будет один, у тебя будет либо два, либо ни одного?

Ответ: Выбор

 

Загадка : Какие из следующих слов не принадлежат к группе и почему? КОРСЕТ, КОСТЕР, СЕКТОР, ЭСКОРТ, СУДЫ

Ответ: Суды.Все остальные являются анаграммами друг друга.

 

Загадка : Я в четыре раза старше своей дочери. Через 20 лет я буду вдвое старше ее. Сколько нам сейчас лет?

Ответ: Мне 40, моей дочери 10.

 

Загадка : Два отца и два сына в машине, но в машине всего три человека. Как это возможно?

Ответ: Это дедушка, отец и сын.

 

Загадка : кто ходит на четырех ногах утром, на двух ногах в полдень и на трех ногах вечером?

Ответ: Человек

 

Загадка : Я вижу это, оно видит меня, оно похоже на меня, я кажусь таким же, как оно.Я разговариваю с ним, но он не отвечает мне. Что это?

Ответ: Фотография (моя)

 

Загадка : Мальчик бежит домой, делает три поворота налево и обнаруживает, что его ждут двое мужчин в масках, что случилось?

Ответ: Мальчик играет в бейсбол. Он бежит к домашней тарелке и находит ожидающих его судью и ловца.

 

Загадка : Что имеет отверстия, но удерживает воду внутри?

Ответ: Губка.

 

Загадка : Я присутствую в начале любви; начало жизни, а я конец зла. Что я?

Ответ: Буква «Л».

 

Загадка : Что находится перед вами, но вы можете это видеть?

Ответ: Ваше будущее

 

Загадка : Кормит всех, но не может есть?

Ответ: Ложка

 

Загадка : Кто имеет пятнадцать бриллиантов, но не богат?

Ответ: Колода карт.

 

Загадка : Как можно уронить стакан на твердый пол, не разбив его?

Ответ: Твердый пол стеклом не разбить!

 

Загадка : Что имеет много ключей, но не может открыть ни один замок?

Ответ: Пианино

 

Загадка : Что черное, когда чистое, и белое, когда грязное?

Ответ: классная доска

 

Загадка : Что становится больше, когда отнимается больше?

Ответ: Дырка

 

Загадка : Я иду вверх, я спускаюсь, но я стою сталью.Что я?

Ответ: Лестница

 

Загадка : Меня находят в носках, шарфах и перчатках; и часто в лапках шаловливых котят. Что я?

Ответ: Пряжа

 

Загадка : Что нельзя положить в кастрюлю?

Ответ: Его крышка

 

Загадка : Я могу показать вам числа, я могу издать звук. Ты не любишь меня по утрам, я нужен тебе по ночам. Что я?

Ответ: Будильник.

 

Загадка : Что поднимается и опускается, но не движется?

Ответ: Лестница

 

Загадка : Он короче, чем остальные, и когда он счастлив,

Ответ: Большой палец

 

Загадка : Если два — это корпорация, а три — толпа, то что такое четыре и пять?

Ответ: Девять

 

Загадка : Какие три числа, кроме нуля, дают одинаковый результат при сложении или умножении?

Ответ: Раз, два и три

 

Загадка : Из одного человека получается два.

Ответ: Зеркало.

 

Загадка : Что тяжелее: много кирпичей или много перьев?

Ответ: Ни то, ни другое — они оба весят тонну.

 

Загадка : Летом холодно, а зимой горит, летом одиноко, а зимой в компании. Что я?

Ответ: Камин.

 

Загадка : Маленькая девочка идет в магазин и покупает двенадцать яиц.Когда она идет домой, все, кроме трех, сломаны. Сколько яиц осталось неразбитыми?

Ответ: Три

 

Загадка : Свистит, стонет, жужжит, скрипит. Что это?

Ответ: Ветер

Я Зоя, 23 года. Я Театролог, я очень люблю искусство и в основном писательство. Я путешественник, базирующийся в Греции, Афинах, я не могу жить без улыбок, вкусной еды и, конечно же, домашних животных. В свободное время я смотрю старые классические фильмы или гуляю по центру города.Идеальный мир для меня содержит свободу, любовь, флирт и бесплатные билеты на самолет.

Последние сообщения Зои Цадари (посмотреть все)

Была ли эта статья полезной?

ДаНет

15+ Проверьте свою смекалку с помощью сложных и трудных загадок для взрослых 2022

Проверьте свою смекалку, решая эти загадки для взрослых. Похоже, мы не из тех, кто действительно перерастает свою любовь к загадкам независимо от возраста. Мы делимся с вами некоторыми из простых и сложных загадок для развлечения. Наслаждайтесь вызовом с семьей и друзьями.

Умница Загадки для взрослых

Докажите свою сообразительность, решая эти хитрые и умные загадки для взрослых. Загадки, как правило, сложны, но это хороший способ проверить себя и убедиться, что ваш мозг все еще работает над такими вещами, как загадки для взрослых. Когда вы застряли, просто остановитесь и подумайте. Сделайте паузу, отдохните и подумайте. В конце концов у тебя все получится!

Делитесь этими шутками? ❤️️

Пожалуйста, добавьте ссылку на эту статью. Ваша поддержка помогает нам писать больше интересных статей для вас и всех любителей анекдотов 🙂

Прочитано: очень сложных загадок (с ответами)


Какое пальто всегда мокрое, когда вы его надеваете?

Слой краски.


Люди делают меня, спасают меня, изменяют меня, воспитывают меня. Что я?

Деньги.


 Говорят, что я разоряю врачей, иногда я кислая, иногда сладкая, меня можно есть и пить. Что я?

Яблоко.


Почему маленький мальчик закопал свой фонарик?

Потому что батарейки сели.


Как сделать из шести нечетное число?

Удалите букву S, и вы получите IX, что равно 9 римскими цифрами.


Что разбивается, но никогда не падает, и что падает, но никогда не разбивается?

День и ночь.


 Как узнать, что Юпитер, Сатурн, Уран и Нептун собираются пожениться?

Потому что у всех есть кольца.


Что имеет лицо и две руки, но не имеет ни рук, ни ног?

Часы


 Я никогда не был, но всегда буду. Меня никто никогда не видел, но все знают, что я существую.Я даю людям мотивацию становиться лучше каждый день. Что я?

Завтра


 Загадки для взрослых

Возраст — не что иное, как число. Независимо от того, сколько нам лет, мы все равно будем наслаждаться хорошими и чистыми загадками для взрослых. Сделайте игровой вечер веселее и увлекательнее с помощью всех этих загадок для взрослых.


Я видел лодку, полную людей, но на лодке не было ни одного человека . Как это возможно?

Все они были женаты.


Я дважды появляюсь в вечности, и я всегда в поле зрения. Что я?

Буква «Т»


Мальчик и инженер ловили рыбу. Мальчик — сын инженера, но инженер — отец мальчика. Тогда кто инженер?

Инженер — мать мальчика.


Джефф моложе Родни, но старше Дебби. Ларри старше Эрики, которая старше Джеффа.Родни старше Ларри. Кто средний ребенок?

Эрика.


Король, королева и двое близнецов лежали в большой комнате. Как нет взрослых в комнате?

Это все кровати!

Вы всегда найдете меня в прошлом, я могу быть создан в настоящем, но будущее никогда не сможет меня испортить. Что я?

История.

Что имеет четыре ноги, как у собаки, два глаза, как у собаки, выглядит так же, как собака, но не является собакой?

Изображение собаки!

Мальчик и инженер ловили рыбу.Мальчик — сын инженера, но инженер — отец мальчика. Тогда кто инженер?

Инженер — мать мальчика.

У бедняков есть. Богатым людям это нужно. Если ты съешь его, ты умрешь. Что это?

Ничего.


Короткие Загадки для взрослых

Вы знаете, как говорят: «ПОЦЕЛУЙ». Говорите кратко и просто. Чем короче загадка, тем больше у вас шансов правильно ее пересказать. Так что соглашайтесь на короткие загадки для взрослых, если вы хотите быстро задать вопрос и ответить на него со своими друзьями и семьей.


У кого тринадцать сердец, но нет других органов?

Колода карт.


Что растет, но никогда не падает?

Ваш возраст.


Ответ  Я даю да, но я имею в виду нет. Что за вопрос ?

«Вы не возражаете?»


Почему мы не можем сфотографировать человека с деревянной ногой ?

 Деревянная ножка – это не камера.


Как можно бросить сырое яйцо на бетонный пол, не разбив его?

Бетонный пол от яйца не треснет!


Сколько времени, когда слон сидит на заборе?

Пришло время починить забор.


Резюме

Ну вот! Это ваш список загадок для взрослых, которые одновременно и смешные, и сложные. Но если вы хотите подняться на ступеньку выше, вы можете это сделать. Создавайте свои собственные загадки для взрослых и вперед.Подумайте об объекте и подумайте о характеристиках, которые могут относиться к объекту, о котором вы думаете. Сделать это хорошо и сложно.

Но эй, мы не все о загадках. Мы тоже готовы к шуткам! Проверьте все эти списки шуток, которыми вы можете поделиться с семьей и друзьями, чтобы посмеяться над ними.

50 головоломок для взрослых

Вот несколько головоломок для взрослых с ответами, которые помогут озадачить ваших друзей и родных. Разгадывайте их с помощью этих головоломок и проверяйте, смогут ли они правильно на них ответить.Мы надеемся, что эти загадки сделают ваш день.


Головоломки-головоломки

Загадка: Какое слово в английском языке означает следующее: первые две буквы означают мужчину, первые три буквы означают женщину, первые четыре буквы означают великую, а весь мир означает великую женщину. Что за слово?

Ответ : Героиня

Загадка: Ты измеряешь мою жизнь часами, а я служу тебе, истекая. Я быстрый, когда я худой, и медленный, когда я толстый. Ветер мой враг.

Ответ : Свеча.

Загадка: Как может 8 + 8 = 4?

Ответ: Когда вы думаете о времени. 8 утра + 8 часов = 4 часа.

Загадка: Какое английское слово состоит из трех последовательных двойных букв?

Ответ : Бухгалтер.

Загадка: A — брат B. B — брат C. C — отец D.Итак, как D связано с A?

Ответ : A дядя D.

Загадка: Какое слово в словаре всегда пишется неправильно?

Ответ : «Неправильно».

Загадка: Что видно в середине марта и апреля, чего нельзя увидеть в начале или в конце любого месяца?

Ответ : Буква «R».

Загадка: Вы видите лодку, полную людей. Он не затонул, но когда вы снова посмотрите, вы не увидите ни одного человека на лодке. Почему?

Ответ : Все люди были женаты.

Riddel: Что такое курица 3/7, кошка 2/3 и коза 2/4?

Ответ: Чикаго

Загадка: Врач и водитель автобуса влюблены в одну и ту же женщину, привлекательную девушку по имени Сара. Водителю автобуса пришлось отправиться в длительную автобусную поездку, которая продлится неделю. Перед уходом он дал Саре семь яблок. Почему?

Ответ: Одно яблоко в день избавит от доктора!

Загадка: Я покинул свой лагерь и прошел на юг 3 мили.Затем я повернул на восток и прошел пешком 3 мили. Затем я повернул на север и прошел пешком 3 мили, когда я наткнулся на медведя в своей палатке, который ел мою еду! Какого цвета был медведь?

Ответ: Белый, так как единственное место, где вы можете это сделать, это Северный полюс, а белые медведи — единственный вид медведей, которые живут там.

Загадка: Женщина стреляет в своего мужа. Затем она держит его под водой более 5 минут. Наконец, она вешает его. Но через 5 минут они оба выходят вместе и наслаждаются прекрасным ужином вместе.Как это может быть?

Ответ: Женщина была фотографом. Она сделала снимок мужа, проявила его и повесила сушиться.

Загадка: Что особенного в этих словах: работа, полироль, трава?

Ответ: Они произносятся по-разному, когда первая буква заглавная.

Загадка: Меня вытащили из шахты и заперли в деревянном ящике, из которого меня никогда не выпустят, и тем не менее меня используют почти все.Что я?

Ответ: Грифель карандаша

Загадка: Человек стоит на одном берегу реки, его собака — на другом. Мужчина зовет свою собаку, которая тут же переправляется через реку, не промокнув и не используя мост или лодку. Как собака это сделала?

Ответ: Река замерзла

Загадка: Меня нельзя купить, но можно украсть одним взглядом. Я бесполезен для одного, но бесценен для двоих. Что я?

Ответ: Любовь

Загадка: Переверни меня на бок, и я стану всем.Разрежь меня пополам, и я ничто. Что я?

Ответ: Число 8

Загадка: Неважно, как мало или как много ты меня используешь, ты меняешь меня каждый месяц. Что я?

Ответ: Календарь

Загадка: У чего есть города, но нет домов; леса, но нет деревьев; и вода, а рыбы нет?

Ответ: Карта

Загадка: Красный дом сделан из красного кирпича. Синий дом сделан из синих кирпичей.Желтый дом сделан из желтых кирпичей. Из чего сделана теплица?

Ответ: Стекло

Загадка: В английском языке есть слово, в котором первые две буквы обозначают мужчину, первые три буквы — женщину, первые четыре — великого мужчину, а весь мир — великую женщину. Что за слово?

Ответ: Героиня

Загадка: У меня в руке две монеты, только что отчеканенные.Вместе они составляют 30 центов. Один не копейка. Что за монеты?

Ответ: Четвертак и пятак

Загадка: Что труднее поймать, чем быстрее бежишь?

Ответ: Ваше дыхание

  Загадка: Что не имеет рук, но может постучать в твою дверь, и если это так, тебе лучше открыть?

Ответ: Возможность

Загадка: Что вы выбрасываете, когда хотите использовать, и принимаете, когда не хотите?

Ответ: Якорь

Загадка: Сколько букв в алфавите?

Ответ : В слове «алфавит» 11 букв

Загадка: Я могу быть коротким и иногда горячим.Когда меня показывают, я редко впечатляю. Что я?

Ответ: Характер

Загадка: Чем можно управлять, хотя у него нет колес, разрезанный, но остающийся целым?

Ответ: Мяч для гольфа

Загадка: Стоимость изготовления знает только производитель, Ничего не стоит, если покупать, но иногда обменивать. Бедный человек может отдать ее так же легко, как король. Когда кто-то сломлен, боль и обман гарантированы. Что это?

Ответ: Обещание

Загадка: Что качается, но не катится?

Ответ: Кресло-качалка

Загадка: Стреляй в меня тысячу раз, и я все еще могу выжить.Но одна царапина от меня, и вы обнаружите, что ваши перспективы падают. Что я?

Ответ: Восьмерка

Загадка: Что ты можешь держать в левой руке, но не в правой?

Ответ: Ваш правый локоть

Загадка: Почему буква F подобна смерти?

Ответ: Потому что без него жизнь — ложь, или она делает жизнь ложью.

Загадка; Ты не хочешь его иметь, Но когда он у тебя есть, Ты не хочешь его потерять.Что это?

Ответ: Иск

Загадка: Лифт находится на первом этаже. В лифте четыре человека, включая меня. Когда лифт достигает первого этажа, выходит один человек и входят три человека. Лифт поднимается на второй этаж, выходят 2 человека, входят 6 человек. выходит, но входит 12 человек. На полпути к следующему этажу трос лифта обрывается, он падает на пол. Все остальные умирают в лифте, кроме меня.Как я выжил?

Ответ: Я вышел на первом этаже

Загадка: Угадай следующие три буквы в ряду GTNTL.

Ответ: I, T, S. Полная последовательность — это первая буква каждого слова в предложении.

Загадка: Что нельзя положить в кастрюлю?

Ответ: Его крышка

Загадка: У Арнольда Шварценеггера есть длинная. У Майкла Дж. Фокса короткая.Мадонна своим не пользуется. Билл Клинтон всегда использует его. Папа никогда не использует его. Что это?

Ответ: Фамилия

Загадка: Число 8 549 176 320 уникально. Что в нем такого особенного?

Ответ: Это единственное число, в котором все цифры расположены в алфавитном порядке.

Загадка: Найдите число меньше 100, которое увеличивается на одну пятую своего значения при перестановке цифр.

Ответ: 45 (1/5 от 45 = 9, 9 + 45 = 54)

Загадка: Какая буква идет следующей в следующей последовательности? Д РМ Ф С Л Т_

Ответ: Д – каждая буква соответствует одной ноте в диатонической музыкальной гамме: До, Ре, Ми, Фа, Соль, Ля, Ти, До

Загадка: У владельца ранчо было 500 коров, и он заразил их всех одним выстрелом.Как он это сделал?

Ответ; Панорамное фото

Загадка: Женщина стреляет в своего мужа, а затем держит его под водой в течение пяти минут. Далее она вешает его. Сразу после этого они наслаждаются прекрасным ужином. Объяснять.

Ответ : Она сфотографировала его и проявила в своей фотолаборатории.

 Загадка: В 1990 году человеку было 15 лет. В 1995 году тому же человеку было 10 лет. Как это может быть?

Ответ: Человек родился в 2005 году Б.С.

Загадка: Вы находитесь в месте под названием Мир Уолли, где действует только один закон. Зеркало есть, а отражения нет. Есть пицца с сыром, но не с колбасой. Перец есть, а соли нет. Есть дверь, но нет ни входа, ни выхода. Что такое закон?

Ответ: Каждое слово в Wally’s World должно содержать двойные буквы.

Загадка: Я бываю разных форм и размеров. Часть меня изогнута, часть прямая. Вы можете поставить меня куда угодно, но есть только одно правильное место для меня.Что я?

Ответ: Кусочек головоломки

Загадка: Каждый день я ослабляю всех мужчин на несколько часов. Я показываю тебе странные видения, пока тебя нет. Я беру тебя ночью, днем ​​беру тебя обратно. Никто не страдает из-за меня, но страдает из-за моей нехватки. Что я?

Ответ: Сон

Загадка: У меня нет ни глаз, ни ног, ни ушей, и я помогаю двигать землю. Что я?

Ответ: Дождевой червь

Загадка: Какая шина не двигается, когда автомобиль поворачивает направо?

Ответ: Запасное колесо

Загадка: У девочки столько же братьев, сколько и сестер, но у каждого брата вдвое меньше братьев, чем сестер.Сколько братьев и сестер в семье?

Ответ : Четыре сестры и три брата.

Загадка: Что исчезает, как только вы произносите его имя?

Ответ : Тишина

Загадка: Вы находитесь в месте под названием Мир Уолли, где действует только один закон. Зеркало есть, а отражения нет. Есть пицца с сыром, но не с колбасой. Перец есть, а соли нет. Есть дверь, но нет ни входа, ни выхода.Что такое закон?

Ответ: Каждое слово в Wally’s World должно содержать двойные буквы.

Загадка: Как число четыре имени пять?

Ответ : IV, римская цифра четыре, которая составляет «половину» (две буквы) слова пять.

Загадка: Солнечные часы имеют наименьшее количество движущихся частей среди часов. У кого больше всего?

Ответ: Песочные часы с тысячами песчинок

Загадка: У меня есть ключи, но нет ни замков, ни места, ни комнат.Вы можете войти, но вы не можете выйти наружу. Что я?

Ответ : Клавиатура

Загадка: Что намокает при сушке?

Ответ : Полотенце.

Загадка: Во-первых, подумайте о цвете облаков. Затем подумайте о цвете снега. Теперь подумайте о цвете яркой полной луны. А теперь ответьте быстро, что пьют коровы?

Ответ : Вода.

Загадка: Что приходит раз в минуту, дважды в мгновение, но никогда в тысячу лет?

Ответ : Буква «М.

Загадка: В какой месяц люди меньше всего спят?

Ответ: Февраль (в феврале обычно меньше ночей).

Загадка: Братья и сестры, у меня нет никого, кроме отца этого человека, сына моего отца. Кто этот человек?

Ответ: Этот человек мой сын


Поддержание вашего мозга в тонусе и активности так же важно, как и ваше тело. Головоломки и загадки — лучшее, что нужно для здоровья вашего мозга.Есть много типов головоломок, чтобы проверить вашу способность решать проблемы и критическое мышление.

Самое главное, что вы должны помнить, это то, что у головоломки может быть более одного правильного ответа. Если вы считаете, что наш ответ неверен или требует каких-либо изменений, сообщите нам об этом в разделе комментариев ниже. Мы с нетерпением ждем ваших добрых отзывов, комментариев и предложений.


Не забудьте поделиться им со своими друзьями и семьей, чтобы узнать, смогут ли они его получить, а пока у нас есть еще один для вас.

Веселые и умные загадки для взрослых

Развлечения

Опубликовано Адамом Фрейлихом, Обновлено 21 августа 2019 г.

Считаете себя мастером загадок, которому нужны весёлые и умные загадки для взрослых? Вы постоянно пытаетесь выяснить, как стимулировать свой ум? Будь вы опытным профессионалом в области загадок, таким как Загадочник из «Бэтмена», или просто любителем убить время и растянуть старую мозговую мышцу. Загадки — отличный способ держать ум в тонусе.Они также будут ставить в тупик ваших друзей и коллег, когда вы находитесь в городе или в командировке. Вот список 20 лучших забавных и умных загадок для взрослых. Просто пообещай, что не совершишь загадочных преступлений в Готэме.

Подписаться на List25


https://www.getriddles.com/short-riddles/

A: Удалите букву S, и вы получите IX, что равно 9 римскими цифрами.

https://www.getriddles.com/short-riddles/ https://www.getriddles.com/short-riddles/

О: Нет, все снаружи.

https://www.getriddles.com/short-riddles/

A: Они предпочли яблоки, а не финики.

https://www.getriddles.com/short-riddles/ https://www.getriddles.com/short-riddles/ https://top-funny-jokes.com/clever-riddles/

A: Лошадей звали вторник.

https://top-funny-jokes.com/clever-riddles/

A: Еще один после этого уже не пустой.

https://top-смешные-шутки.com/умные-загадки/ https://top-funny-jokes.com/clever-riddles/ https://top-funny-jokes.com/clever-riddles/ https://www.riddles.com/riddles-for-adults

A: Только один, «Когда я шел в торговый центр, я встретил мужчину…»

https://top-funny-jokes.com/clever-riddles/ https://top-funny-jokes.com/clever-riddles/ https://top-funny-jokes.com/clever-riddles/ https://top-funny-jokes.com/clever-riddles/ https://top-funny-jokes.com/clever-riddles/ http://www.theactivitydirectorsoffice.com/RiddlesWithAnswers.pdf

А: Петухи не могут нести яйца



Фото: Избранное изображение — Shutterstock, 1. pixabay.com (общественное достояние), 2. pixabay.com (общественное достояние), 3. pixabay.com (общественное достояние), 4. pixabay.com (общественное достояние), 5. pixabay .com (общественное достояние), 6. pixabay.com (общественное достояние), 7. rajaraman sundaram, DARK CAVES, BELUM, AP — panoramio, CC BY 3.0, 8. UnknownUnknown author, Mens Coats 1872 Fashion Plate, Public Domain, 9 Имре Солт, торговый центр Mercato, 5 июня 2007 г. Фото 2, CC BY-SA 3.0, 10. pixabay.com (общественное достояние), 11. pexels.com (общественное достояние), 12. pixabay.com (общественное достояние), 13. pixabay.com (общественное достояние), 14. pixabay.com (общественное достояние). ), 15. Рудольф А.Фуртадо, Пара на велосипеде в Париже., Public Domain, 16. pixabay.com (Public Domain), 17. Лукас Кранах Старший, создатель QS:P170,Q1, Кранах Старший Адам и Ева, общественное достояние, 18. JJ Harrison ([электронная почта защищена]), Oryctolagus cuniculus Tasmania 2, CC BY-SA 3.0, 19. pexels.com (общественное достояние), 20. pixabay.com (общественное достояние)

математических загадок для взрослых | Учиться.ком

Геометрические фигуры

У вас есть идеальный куб. Можете ли вы разделить свой куб на более мелкие кубы, которые имеют разные объемы?

  • Решение: Нет. Кубы всех объемов не могут правильно поместиться внутри большего куба. По мере того, как кубики становились все меньше и меньше, были бы пустые места, и у вас не было бы возможности заполнить эти места. В конце концов, кубики станут слишком маленькими, чтобы продолжать.
Подсказка:

Вы можете использовать этот урок по трехмерным фигурам, чтобы узнать о характеристиках этих типов фигур.Этот урок по нахождению объема куба также поможет вам лучше понять формулу, используемую для расчета объема этой трехмерной фигуры.

Вероятность

Однажды утром вам нужно найти носки, но у вас сломался свет. Вы знаете, что в вашем ящике есть 10 фиолетовых и 10 оранжевых носков, но все они перепутаны. Они абсолютно одинаковые, кроме цвета. Сколько носков вам нужно выбрать, чтобы убедиться, что у вас есть подходящая пара?

  • Решение: Вам нужно выбрать только 3 носка.Поскольку в ящике есть только два цвета носков, у вас обязательно будет подходящая пара, если вы возьмете 3 из комнаты.
Подсказка:

Сфокусируйтесь на улучшении своего понимания этого предмета с помощью этого урока о вероятности зависимых и независимых событий. Это видео — отличный способ быстро пройтись по правилам, регулирующим вероятность.

Word Problems

Вы режете пирог. Сначала вы разрезаете его на две части.При следующем разрезе вы производите 4 детали. В результате третьего разреза получается 7 частей. На сколько частей можно нарезать семь ломтиков?

  • Решение: Решите эту задачу, учитывая правило, установленное срезами. Из 1 разреза получается 2 детали. Второй разрез дает 4, а третий — 7. Таким образом, каждый разрез увеличивает количество частей на номер разреза (1, 2 и 3).

Добавить комментарий

Ваш адрес email не будет опубликован. Обязательные поля помечены *

2019 © Все права защищены. Карта сайта